You are on page 1of 61

. , ..

14
Di ffe ren tia l EquatiQn I

The equa tions whic h invol ving derivatives of unkn


own In this chapter we solve only ordin ary d_if~erential
or?er
variable is calle d differential equat ions. There are two types equations. The students are advised to know this m
iques of solvm g
of differential equa tion- to have a better understanding of the techn
}. Ordi nary differential Equa tion ordinary differential equation.
2. Parti al differential equa tion

.,
ORD INA RY DIF FER E~T IAL EQU,ATI QN':

or more
A differential equa tion involving derivatives of one ... ) d 2y .
to singl e indep ende nt (_111 dx 2 + sm y = x
depe nden t varia ble with respe ct
variable is calle d ordin ary differential equat ion. orde r= 2
For Exam ples: . d3 y dy d 2y
2
. x-=dy x (")
2
d y x-= dy sm . x (1v) y dx3 + 3 dx dx2 = 0
d y sm
. --+ 11 --+
(1) 2 orde r= 3.
dx 2 dx dx dx
Deg ree of the Diff eren tial Equ atio n
(m
dx

- y-
" ') dy- = -
1- x
2
2 2
d y
( iv) -+
dx 2
dy
5-+ 6y =O
dx
The powe r of the highe st derivative Such that diffe
equation should be free from radical sign
called the degre e of the differential equa
and
tion.
renti al
fract ion is

For Exam ples:


Jx )
+ (d
3
2 . dy
(v) :2; =x . d y -= x
(1) - - +sm x
dx 2 dx
Ord er of the Diff eren tial Equ atio n degr ee= 1
The pow er of a high est deriv ative invo lv~d
in t?e 2
d the orde r of d1fferent1al
diffe renti al equa tion is calle (ii) d ; +sin (dy) =x
dx dx
equa tion. degr ee= can't deter mine d.
For Exam ples:
2 ... . d-y ') d
_r
2 + cosx dx
(111) smx --
( )3 + 2y = cot x
(i) d y +sin (dy) =x dx
dx2 dx
degr ee= I
orde r= 2
3
d 2 d2y
(ii) [1 + ( ~) ] = dx'

orde r= 2 degr ee= I


659
Remark..q: .
t
I. The order of the differential equation always exists but somewhere degree does not exiS .

2. If '°
£...
d"J"( Y) = oo n E N is finite then we determined order.
dx"

DIFFE~EN TIAL EQUATION S OF THE FIRST ORDER AND FIRST DEGREE

(i) Variable separable equations


The differential equation of first degree and first order is
defined as, (ii) Homogeneous differential equations
y' = f(x, y) (iii) Exact defferential equations
The differential equation of the first order and first (iv) Linear differential equations
degree can not be solved in each case. Method of solving ( v) Integrating factor.
the different standard form of first order and first degree (vi) Reducible to homogeneous
differential equation are given below: (vii) Reducible to linear differential equation.

(1) VARIABLE SEPARABL E EQUATION S

If the differential equation of first order and first degree Solution: We are given that,
can be reduced into the formJ;(x) dx= /2(y) dy dy
=
log(:)= 3x + 4y or -dx e3x- 4y

Then, JJi (x) dx =f f 2 (y) dy + C


dy
This is called separation of variable method. - = e3X· e4Y or e-4Ydy =e3Xdx
Example-I. Solve the differential equation, dx
on Integrating, we get,
dy = fr - Y + x 2 e-Y
dx fe -4 Y dy = fe 3
x dx + C
Solution: We are given that, the differential
e-4y e3x

= tr - Y + x 2e-Y
- - =-+C
equation dy -4 3
dx
3e-4Y + 4e 3X = C'
dy = fr . e-Y + x2 e-Y where, C' is an integration constant.
dx
Example-3. Solve, the differential equation.
dy = (fr+ x 2) e-Y
dx (] + x2 ) dy = (1 + y2)
dx
dy = (fr+ x 2 ) dx Solution: We are given that the_ d ifferential
e -y equation,
On Integrating both sides, we have,
(1 + x2) dy = (1 + y2)
f eY dy = f (ex +x )dx+C 2 dx
dx dy
x3 or --2
= --
eY= ex +-+C 1+ x l+ y2
3
On Integrating both sides, we get
where, C is an arbitrary constant of integration.
Example-2. Solve, the differential equation,
J_!Q_-J
1 + y2 -
dx +C
l+x2
log ( ; ) = 3x + 4y or tan- 1y = tan- 1 x + C
where, C is an arbitrary constant of integration.
-----::-- .--:-=-- -------- ------~
DIFFERENTIAL EQUATION

. equations in the form of the variable separable,


1 erential
Remark: Transformation of some d'ffi
661

dy
as, dx = f(ax + by) =f (ax+ by + c)

Solution: We are given that the differential


.
Example-4. Solve, the dziffierentz'a/ equatwn,
dy equation,
dx = (4x + y + 1)2 x + / -1
2
y dy
- - + --.::,__-- ==0
Solution: We are given that the differential X dx 2x2 + 2 y2 + ]
equation, Put x2 + y 2 == t
On differentiating, we get
dy
dx = (4x + y + 1)2 dy dt
2x+2y - == -
dx dx
Put 4x + y + I = t
On differentiating both sides, we get dy == _1 (!!!.... - 2x)
= dt or dx 2y dx
4 +dy
dx dx
Putting the value of dy in the given differential
dx
Put the value of dy in the given differential
dx equation, we have
equation, we get 1 dt t -1
---1+-=0
dt 2x dx 2t + 1
- - 4 = t2
dx 1 dt 3t
--==-
or 2x dx 2t +1
dt
- =t2+4
dx
or ~ dt = 2x dx
3t
_!!!_=dx
2
t +4
or ~ + _!_)dt = 2T. dx .
On integrating, we get ( 3 3t
On Integrating both sides, we get
ft _!!!_
2
= f dx+C
J (½+ ;t )dt
+4
= 2 Jxd\' + C
or ½tan- (f) =x + C
1

or½ran-1( 4x+:+l ) - x+ C or 32( x-? + _v-., ) + 3I log(x-, + y-), = x2 + C


where C is an arbitrary constant of integration. where C is arbitrary constant of integration.
Example-5: Solve that differential equation,
2
l. dy + x+ y2 - 1 == o
2
X dx 2x + 2l + ]
(II) HOMOGENEOUS DIFFERENTIAL EQUATIONS:
dy = cj>(.r. y)
t 0rd dx \jJ (.r, r )
The differential equation of firS 10 0 f~~t
er a?~
degree is said to be homogeneous if it can be put e where, <!> and \j/ are homogeneous ti.1;1ction ·111 ,\,. y.
form
f.6~6~2:___ _ _ _ _ _ _ _ _ _ _ _ _ _ _ _ _ _ _
For Example:
:.:-====--:;~~~~=:====-M~;:ATHe..,Alles
Example-7. Solve the homogeneous d ~
,:
dy + X2 xy equation,
= 2
dx x + y2
Put y = vx in homogenous equation, we get
dy dv Solution: We are given that the homogenous
= v+x-
-
dx dx differential equation,
and equation is reduced to variable and seperable form.
Example-6. Solve: (x 2 -y2) dx-xy dy = 0
Solution: We are given that the homogeneous
(I + ~ dx + e'"'(1- ; }y - 0

differential equation, dy __ -(1 + exty) dy


(x2 - y2) dx - xy dy = 0 or ) or - =
dx exly ( y; X dx
dy dv.h.
Put y = vx and - = v + x - m t e given
' dx dx dx dv
differential equation, we get Put x = vy and - = v + y- in the given
dy dy
2 2 2
dv
v +x- = x -v2 x xdv
- differential equation, we get
dx vx dx
2 1 -(1 + e v ) dv -(1 - v)
1-v --- = or v + y - =
=---v dv
v+y- e v (1 - v) dy 1 + e- v
V
dy
dv 1- 2v2 V
x- = dv dv (v-1) dv
dx V 1- 2v2 or y- = ·- v or y - =
dy (I+ e-v) dy
dx 1 dt dx
=- - -- =-
X 4 t X dy 1 + e-v dv
or =
On Integrating, we get, y ve-v -1
1 .
On Integrating both sides, we get,
- 4 Iog t = log x + log C
log y= -log(v + ev) + log C.

-
2 +-I
(1-2v ) 4 =x C
(
1-2
y2 )+!=xC.
4 .
or log y- -log(; +e,1' )+loge
7 C
I or y=
(x2:--2y2}4 =x3C X + exly
y
where, C is an integration constant.
where, C is the arbitrary constant of integration.

(111) EXACT DIFFERENTIAL EQUATIONS:


The differential equations of the form,
M(x, y) dx + N(x , y) dy = 0
is said to be exact if there is a function U(x, y), such that,
du = Mdx == Ndy and solution is f ~ +f
( y constant)
Ndy
l___J =C
(not containing x )

Remark: The necessary and sufficient condition for the differential equation to be exact iff
aM aN
ay = a;
D1FfERENTIAL EQUATION
663
Example-8: Solve
Example-to. Solve, the differential equatio~,
y2dx + 2.xy dy = O
(2x3 + 3y2x - 7x) dx + (Jx2y + 2y3 - By)dy - 0.
Solution: We are given that the differential equation Solution: We are given that the differential equation
y2 dx + 2xy dy = 0
(2x3 + 3yx - 7x) dx + (3x2y + 2y3 - Sy) dy =0
On comparing this equation with
On comparing this differential equation with,
M dx + Ndy = C, we get
M dx + N dy = C, we get
M= /2 M= 2x3 + 3y2 x - 7x
and N= 2xy
and N = (3x2 y + 2y3 - Sy)

Now,
aM aN 2JM aN
ay = 2y = ax Now, -· - = 6yx=-
2Jy ax
Hence, the given differential equation is exact. The Thus, The given differential equation is exact. and
required solution is 'given by,
The required solution is given by,
f Mdx + f Ndy = C f [y is constant]M dx + f [not contains 'x]N dy = C
or f y2 dx + f 2xy dy = C or .xy2 + xy2 =c or f (2x + 3y2x -7x) dx+ f (2y3 -8y) dy = C
3

or
4
where, C is an integration constant. x 3 2 2 7 2 y4 4 2
or - +- y X --x + - - y =C
2 2 2 2
Example-9: Solve:
where, C is an arbitrary constant of integration.
2
(y2 exl + 4x3 ) dx + (2xye xy - 3y2 ) dy =0 Example-11. Solve the differential equation,
Solution: We are given that the differential equation dy y cos x + sin y + y =
- + -=--------- 0
2 2 dx sinx+xcos y+x
(y2exy + 4x3 ) dx + (2xy exy -3/) dy =O
Solution: We are given that the different ial
On comparing this differential equation with equation,
M dx + N dy = C, we get dy y cos x + sin y + y
-+------=O
M= y2exy + 4x3
2
dx sin + X cos y + X
X

or (y cos x + siny + y)dx- (sin x + x cosy+ x) dy = O


2 2
and N = 2xy exy - 3y , On comparing this differential equation with, !vfdx
+ Ndy =C, we get
Now,
aM --
- 2 ye xy2 + 2xy3e.x = 2JN
:.. M= y cos x + siny + y
ay ax
and N = -(sin x + x cosy+ x)
Thus, The given differential equation is exact. and
The r!;!quired solution is given by, Now,
aM = cos x + cosy
-a aN
+ 1= -
f M dx+ f N dy = C y ax
'Fhus, The given differential equation is exact. and

or f (y2ex)'2 + 4x3) dx + f (-3y2) dy = C The required solution is given by,

e xy
2
X
4
3 - C
f [y is constant]M dx + f [not containing x] Ndy = c
2 _+4--Y -
or y
or f (y cos x + :,in y + y) dx + O = c
2 4
y

eX)'
2
+ X4 - y
3=C or y sin x + x sin y + xy = C
or
Where, C is an arbitrary constant of integration. where C is an arbitrary constant of integration.
_6 ..: .6 ... ..: 4~ -- -- -- -- --
-- -- -- -- -- -- -- -- -- -- -- -M
(IV ) LIN EA R DIF FE RE NT IAL :: .:A lli EM A ltc s
EQ UA TIO N ------::.:::
A diff eren tial equ atio n of firs t ord
er and first deg ree is
call ed line ar diff eren tial equ atio n ifit
foll owi ng form,
can be writ ten in the
x·e
tan - 1 y -
-
f tan- y
_ _ _etan
__ Y
I -1
d
l 2 Y+C
+y
dy
I • dx + Py = Q, whe re, P and Q are function of x x . etan -I Y = f te t
dt + C Put tan- ty :::
1
or con stan t.
.
imp 1·1es l
~ dy::: dt
dx 1+ y
2. -d + Px = Q, whe re P and Qar e function ofy -I
y x e tan Y = ti - e' + C
or con stan t. -1
xe tan )' -- tan - e tan-I Y -e tan-I Y +C
ly
wor kin g rule for solv ing the line ar
diff eren tial equ atio n,
or x=tan - I y -1 +Ce -tan y ~

dy +P y= Q whe re C is an arbi trar y con stan t of


dx integration.
Example-13. Solve the differential equ
ation,
Ste p I: I.F = eI Pdx (x + 1) dy - y = e 3x (x + 1)2 .
dx
Ste p II: y(l. F.) = f Q (/.F.) dx + C. Sol utio n: We are giv en tha t the
equ atio n,
differential
Exa mp le-1 2. Solve the differential equ
ation,
(1 + y 2) dx = (tan- 1 y - x) dy. (x + 1) dy - y = e3x (x + 1)2 or
Sol uti on: We are giv en tha t the dx
equ atio n, diff ere ntia l dy 1
- - Y ·- - = e3x (x + 1)
( I + y 2)dx = (tan- 1 y-x )dy dx x+ l
On com pari ng this diff eren tial equ
atio n with the linear
or dy 1+ y2 differential equ atio n,
- - --- =- --
dx tan- 1 y- x
dy +P y= Q
dx tan- 1 y- x dx
or
dy 1+ y2 we get p = --1-, Q = e3x (x + 1)
x+ l
dx tan-I y 1 So, Inte grat ing fact or,I .F.
or = - x·--
dy 1+ y2 1+ y2
On com par ing this diff eren tial equ
atio n with the
= es Pdx = e-J x ~ I dx =_1_
line ar diff eren tial equ atio n, x+ l
Thu s, The requ ired solu tion is give
dx n by,
-+ Px =Q
dy y · (1.F.) = f Q(I.F.) dx + C
whe re P= _1 -,Q = tan - ;
1 + y2 1 + y-
1
y(-1-)= f
1+ X
e3x (l+ x)_ _!_ _dx +C
(1 + X)
So, The Inte grat ing fact or,
3x
y e
l.F. = e I .
Pd\' J_ I -, dy - - =- +C
I.F. =e '+y- 1+ X 3
-I
l.F. = etan y
Thu s, The requ ired solu tion is give
n by, y = (x + l)[ e: + C]
x · (I.F.) = f Q · (/.F.) dy + C whe re, C is the con stan t of inte grat
ion.
DrFF EREN TIAL EQU ATIO N
665
equation,
Examp)e-14. Solve, the diff eren tial dx 2

dy J ~t 2 - I
= e log ( x - I)
sec x - = y + sin x I.F. = e .
dx J.F. = x 2 - I
ar d"ffi •
Solution: We are give n that the line 1 eren tial Thus, The requ ired solu tion is give
n by,
.
equation y (i.F.) = Q(I.F.) d.x + C f
sec x : =y + sin x.
or . ( 2_ I)= J (X2 -
l
1)
(x -1) dx + C
2

y X
dy Y sin x
or -==-+ -
dx sec x sec x or y · (x 2 - I) = Jd.x + C
dy y (x 2 - I) = x + C .
or
or - - cos X . y = sin X cos X where, C is the arbi trar y con stan t of inte grat ion. .
dx . Solv e the diff eren tial equ atio n,
h the line ar Example-16
On com par ing this equ atio n wit 1

differential equ atio n, (1 + X2dy + Y = e1an- x


)
dx
dy diff ere ntia l
We get, - + Py = Q Sol utio n: We are giv en tha t the
dx equation,
P= -co s x
2 dy
tan-I X

and Q = sin x cos x (1 + x ) - + y = e


d.x
So, the Inte grat ing fact or, -I

I.F. = ef Pdx = ef -cos x dy I- etan x


or dx + I + x2 . y - I + x2
I.F. = e-sin x
h the line ar
n by, On com par ing this equ atio ns wit
Thu s, The requ ired solu tion is give
differential equation,
f
y(I. F.) = Q(I .F.) dx + C
dy + Py = Q, we get
y e- sinx = f sin X cos Xe-s in X dx +C dx
- I
I etan x
Ye - sin x = _ (1 + sin x)e- sinx + C p = and , Q =-- -
--?
)
y = cesi nx - (1 + sin x). where, 1 + .-c
1 + x-
of inte grat ion..
where C, is the arb itra ry con stan t . _I_ d,
equation, J I + ,~
Exa mp le-1 5. Sol ve the diff eren tial I.F. = ef
Pdr
=e ·
So, the Integrating factor,
2 dy - I
(x - ] ) - + 2.xy = J I.F. = eran x
dx by.
Sol utio n: We are giv en tha t the
diff ere ntia l Thu s, The requ ired solu tion is giv en
equation , f
y(I. F.) = Q(I .F.) dx + C

(x 2 - 1) dy
dx
+ 2.xy = 1
or y • eran
- I
x =
fe - -?
t,Ul - 1 .r
1
e an
- l
.rd,· + C
l + x-
dy 2x _ _I _
1 I
Put tan- x = t unp 1es, - - , dx =
or dx + x2 - 1 y - x2 - 1 • 1· dt
wit h the line ar l+ .c
On com par ing this equ atio n
differential equ atio n, or y. eran - 1x = f e2r . dt + C

dy +P y= Q, e
21
dx or y • e tun
- I
.r =- +C
I 2
2x d, Q
- - -
p -- - a n 2
X - 1
we get, ? tan - 1·\
x- -1 or y·e
f Pdx
So, the Inte grat ing fact or, I.F. == e
~ ~ - - - - - - - - - - -- ~
MA-rt-te~"llcs

6~6;__ _ _- =
-:::~~ ~
6~ FA CT O R
i!iJI (V ) IN TE G RA TI NG
+ Nd y = C, we get
not exacft M dx
N d'.Y -- 0 isfun
+lyi ction o
f h fonn M dx If
e act by mu 1P
o et ex ng so me M = x 2y- 2xy2,
If the• equabtion ad .
. ca11 ed Integrating factor •
3 2
em .0n is and N= -{ x - 3x y)
then 1t can
x and y, suc h a fun ct1
Rule fo r So.1vi•ng th e D iff er en tia l dM = x2 -4 xy .
k' Now
Wor m g . Which is no t •ac tfi t of the
Ex dy
Eq ua tio ns
Inspection: An Integrating
ac or
Rule I: By
may fo un d ou t by and dN = - 3(x2 -2 .xy )
given equation M dx +N dy =O dx
inspection, Here, M x+ Ny =t :-0
1. x dy + y dx = d(xy) 1 1 --r--
S IF -- _ .. = x3y - 2 2 3
2x y - x y + 3x y2
,., xd y- yd x = d(Xz) o, · · - M x + Ny
X2
... .
1 I
I.F .= 2 2
3 _ x dy~ y dx =d [lo g (; )]
X

becomes
y
I
The given equation is,

4. ; : : ~ ~+nt;)] c-;Jdx-(;,-ndy-0 _
which is an ex ac t differential equat10n. Hence The req
urred
_
~) ~ solution is,
xd2y- yd2 x = d [.2!_ lo g(
X -y
S. X - y

dx + (xeY + 2y) dy = 0
Example-17. Solve : eY
t the differential equation
Solution: We are given tha
eY dx + (xeY + 2y ) dy =
0
the giv en
We need to deterinsmine the solution of X
= log C
pection , -y - 2l og x + 3 lo gy
differential equation by or
eY dx + xeYdy + 2y dy =
0
d(xeY) + d( y) = 0
or
or log y - Jog x
2- Iog C = --
X
d(xeY+ y2) = O
Y
On Integrating, we get
xe Y+ y2 =c
]
[ y3 = --
X
or
constant of integration. log -
where C is are arbitrary tion Cx2 Y
Rule II: If the given equa
geneous and Mx + Ny t;
O,
Mdx ,+ Ndy = 0 is homo
Then
y=-= Cx 2 e Y
. f
= -- .! .. _ . nstant of integration.
Integrating factor, l.F. where C is an arbitrary co = 0 15 0
M x+ Ny If the gi ve
Rule III : dx n eq ua tioOn M d" + Ndy
+ xg (x y) dy ==
ferential equation, the fo rm yf (x y)
Example2-18. Solve the dif3
2y) dy = O
(x y- 2x y) dx - (x - 3x 1 . an Integ ra tm
1s . g factor Provided
ial _
ven th at the di ffe re nt Th en -- -_
M x- N y
n, lution: We are gi
equatioSo Mx - Nyt; 0.
2 3
(x y - 2xy2)dx - (x - 3x
2y )d y = O ) y dx - x( l + xy)
tio n With th e ampJe-19. Solve : (] - xy
Ex ·l
On l co
eq mp
ua arn,ing the gi ve n eq ua
tio dy = o
differentia ve n th at th e differen
ua
n: We arx(e I gi
. So lu tioy) + xy ) dy = 0
equation, (I -x y dx
-
DIFFERENTIAL EQU ATIO
N
_ _ _ _ _ _ _ __ _ __
_;6~6?_7
_ _ _ _ __
q;t~,a~t~io~n~w~it~l1_ __ _ _ _
:.:.~O?,n~c~o~m~p~ar~i~n~g~th;;e~g~iv~e~n;e;
__ ;l,,f - xyl + y
M dx + Nd y - 0, we get and N = 2 (x 2_v2 + x + y1 )
Af= (I -xy )y
rJN
N= -x( I + xy ) -
oM
= Jxy2 + I and - =4xy - I
M x- Ny -:t:-0 The n, or r)x
c)M dN-
=I- 2xy and - = - 1 - 2xy 2
The n, - ":'\y
o dx so _I (rJM _ oN ) = (.xy
+(x y +I) = +_! _=g (v)
+ ! )y Y 2 -
' M rJy rJx
I.F. =
So, the Int egr atin g fac tor, is
Mx-Ny = € ' =e .,g ·.
J ' cl·. 1

So, the fntegrating facto r. LF.


I.F. = --- -:; --; ,1: ___ _
2 2 f.F. = y
xy - x y + xy + x 2Y2 atio n by ·_i··. we get
On Multiply ing the giv en equ
1 (if +y. :) dx .i.. 2(.r y-3 + .ry +
I.F .= -
2xy y )dy =O
wit h _i fdx ~ .Vdy. we
1 On com par ing this eq uation
ntia l equ atio n by - -
Mu ltip lyin g the giv en diff ere 2xy ' get
c)M 3
we hav e, dy = 4X) + 2y

½[~-y]dx-½[~ +x]dy =O c)N


- - = 4X) J +
,
2y
ial equ atio n the req uire d and d(X)
whic~ i_s are exa ct- dif fer ent
solut10n 1s, c)N
c)M

I (2~ -½ Y)dx + J(- 2~ )dy = 0 So, dy dX


atio n. The re-q uire d so luti on
log x - xy - log y = C wh ich is exa ct diff ere ntia l equ
is giv en by,
log ( ; ) - xy = C fM dx +f Ndy = C
sta nt of Inte gra tion .
where, C is the arb itra ry con Md x + Ndy =0 is 2 4 ? 6
Rule IV: If the giv equ atio n
en .::_ L + xy2 + ::.l _ =C
2 6
(aM c)N] 2 4 , \' o
X y
ax is a fun ctio n of x onl y say / (x), - - + .xy- + -· .,- =C
not exa ct if dy
N
or 2 .,
t of lnte-gr:.Hi1..) n.
then wh ere C is the arb itra ry cons t:m o is ...,r'
.\/,f r -r .\'.n =
f f (x) dx Ru le V: lfthc- given e-qu:nio n
I.F. = e ·
the form
aN c)M] x"./ ( myd Y + 11 x,(i ·) + \ J _ 1" l111 ' 1·,L\ +11 ·.l tfrl = L.,
( dx dy is a fun ctio n of Y onl y say g(y
),
I . r. - ., _1 :u1d h. k 3.rtc' >.!i
J, ..t \ tc'n [:,\.
-
T l1en,
L' -
If
M h + k+ l
11 + /i + l
then
f g(y ) dy II

l. f. = e3 . (; 2y + X· + y-1) Ill

+2 x h' + .{ + I
Example-20. Sol ve: (xy + y) dx a ·+h + I
dy 0 and II
==-
t th e dif fer ent i a l /fl
Sol uti on: We are giv en tha _'x-\ -') .tx
Exa mp le-2 1. So/ w · \ '( f f 1-
equation
(.ry3 + y) dx + 2(x2y2 + x +_y4) dy ~ 0 (on
with , x(x i · x-'.i ·'J ,Iv {) ·
1
ntia l equ a Sol uti on: Wtc' artc' g ih' ll
chat th1..• J ifti.· rc-n ti al
On com par ing the giv en dif fere
the equ atio n equ atio n.
' Md x + Nd y = 0, we get
,
MATHEMAT1es
668 ---.:::::
2
y(xy + 2x2y2) dx + x(.iy - x y2) dy = 0 So, the Integrating factor, isl.F. = x- 3y 3
2 3
or(xy2 + 2x2y3) dx + (x y - x y2) dy = 0 1
On multiplying by x"/, we get I.F.=33
X y
(x" + 1/ + 2 + 2xh + 21 +3) dx + (x'' +2/ + I -xh
+ 31 + 2) dy = 0
Comparing this equation with Mdx + Ndy =O, we On multiplying the given equation by __!__
x3y3
get,
(++I)c1x+(-1 _.!_)dy
M= xh+ 1 / + 2 + 2xh +2 / + 3
and N = xh + 2 / + 1 _ x h + 3 / + 2 X y X xy 2 y
dM which is exact differential equation. The required solution
Thus - =(k+2)x'1 + 11+ 1 +2
dy is given by,
(k+3)xh +2/+ 2
and
dN
dx =(h+2)xh +l / +1_(h+3) f (-1
x y
-3_) dx + f -1Y dy = C
2
X
xh +2 / + 2
which implies that, 1
---21og x-log y==C
k+2=h+2 or k= h xy
and 2k+ 6 =-h-3
where C is an arbitrary constant of integration.
or 2k+ 6 + k+ 3 = 0 or k = -3
Hence k=h=--3
(VI) EQUATIO N REDUCIBLE IN !zt()MOGEl')IE()·~s ··:e:9 :9 6;f:tf.i~.:;,?
On comparing the above equation with
dy ax+by + c
The equation of the form, - = , , (where a, dy = ax+by+c
dx a x+b' y +c
dx . a'x+b'y+c '
b, c, a', b', c' are constants)
can be reduced to homogeneous form by taking the a b
following cases: we get --:;:.-
a' b'
Case I: If .!!-.._ -:;:. !?_ Put x=X+h dx=dX
a' b' and y=Y + k and dy = dY
Put x =X+ h
y= Y+k dY Y + k + X +h - 2
and Then
then dx=dXdy= dY dX - Y+k-X-h -4
. . dY a(X +h)+b(Y +k)+c dY _ Y + X + (k + h - 2)
Theequat1on1s dX = a '(X+h)+b' (Y+k)+c' or dX - Y - X + (k - h - 4)
dY aX + b Y + ah + bk + c where k+h-2=0
or dX a ' X + b 'Y + a 'h + b 'k + c k-h-4=0
Let, On solving these equations, we get
ah + bk + c = 0 and a' h + b' k + c' = 0 k=3,h = -I
Incomplete dY Y+X
Now -=--
dy = y +x-2 dX Y-X
Example-22: Solve:
dx y-x-4
Solution: We are given that the differential
dY ( I_
X
+ 1)
equation,
dy = y +x-2
y- x -4
dX = (f -1)
dx
DIFFERENTIAL EQUA TION

Put 669
Y=== vX
dY (Jy + 2x + 4) dx - (4x +6y + 5) dy =O
Then, - === V+X dV dy3y + 2x + 4
dX - or =-----
dX
dx 2(2x +3y) +5
Again, V+X ~::: :::V +I On comparing this differential equation with
dX ~
dy = ax+b y +c
dx a'x +b' y+c '
X dV === V + 1 - V == 2V - y2 + 1
dX V-1 V a b 1
-1 - = -=-
a' b' m
or (2): ~;)+I }v -~ . Now, put 2x + 3y = t in the given diffe renti al
equation,
dy dt
On Integ ratin g, we get- or 2 +3- =- we get
dx dx'
) d
J(2V V-1
- V + 1 dV f; 2 === .!_ ( dt _ 2 ) = t + 4
2t + 5
3 dx
1 2 3t + 12
--lo g (2V - V + 1) -_ log X + log c dt
= +2
2
dx 2t + 5
log(2 V - V 2 + 1) === 2 log X + log C
dt 3t + 12 + 4t + 10
or X2+ 2xY -Y2 =c -=-----
dx 2t + 5
where X=x +l
Y=y -3 dt ?t + 22
Henc e (x + 1)2 + 2 (x + 1) (y- 3 ) _ (y- 3)2 == 0 dx 2t +5

or
Case II: If .:!:_ ==
a'
!?_ = _!__
b' m
a'== am; b' == bm
(7 ~t:;
2
)dt == dx

On Integrating both sides, we 0oet'


dy == ax + by + c
Then,
dx a'x+ b'y+ c' J( 7~t:;2) dt = f dx + C
dy
dx
ax+b y +c
amx +bm y+c' f (~- 7(7,:22) )dt ~ f dx+C
ax+ by+ c
dy 2 9
m(ax + by)+ c'
dx
7t - 49 log (7t + 22) =x + C

Example-23. Solve : (3y + 2x + 4) ~ (?- X+3 y)--9log[ 7(2x +3Y )+?? ] _


dx - (4x + 6y + 5) dy = O or
7 49 · -- -x + C
Solu tion: We are give n that the diffe renti al where C is an arbitrary constant of ·•nteo
e,rat·ions.
equation,
AR FOR M
(VII ) EQU ATI ON RED UCI BLE TO A LINE
Put y ' - " ==t
An equation of the form
( I - II) y-" dy == ~
dy +Py = Qyn . d.\· dr
dx t Can
Where P and Qare the funct ions of x only or conS ant. dy I dt
or y
- 11
- ==--
be reduced to a linea r form as form - d.x (I - n) d.r
First we multi ply the given equa tion by y-n, we get-
Putting the value of ·v- 11 dy in ab . .
y - n dy + py1 - n = Q d.r ove equation. we
dx
670
get I dt I
I dt
- -- +- l = x2
5 dx X
- - -+ Pt =O
(1-11) dx -
:!!_ - ~ t = - 5x 2
dt dx x
- + P( I - 11 )t = Q( I - n)
dx which linear differential equation then,
dv
Example-24. Solve: x dx + Y = x3y 6 I. F. = e
-J ~dx =e-5
x log x == J_
Solution: We are given that the differential equation , xs
The required solution is given by,
dy + y = .x::3y 6
X
dx l . (1.F.) = IQ. ( I.F.) dx + C
or
dy y
-+- 2 -
=x yb or I
t ·- = J-dx+
5x C
2

dx X xs xs
On multiplying this equation by y-6, we get
or _t
x5
f
= x-3dx+ C
y -+-I y -s = x-,,
- 6 dy
dx X -2
. Put y-5 = t or !_ = ~+C
xs -2
On differentiating with respect to 'x' we get
t= --x +Cx5
1 3
or
-6 dy dt -6 dy I dt 2
-S y dx = dx or y dx = -5 dx
or x-sy-s =. -5 x-2 + C
Putting the value of y-6 dy in the above equation, we get 2
dx where C is an arbitrary constant of integration.

DIFFE RENT IAL 'EQUA TION- .OF FIRST ORDE R .BUT.. NOT OF .,. Fl~RST DEGREE
- - ---~- '- - ~ ~-
.

The first order differential equation of degree n > 1 is, Metho d-I: Equat ion Solva ble for P

+(: J +. .+(: J
Let PrJi1 + P 1p" - 1 + . . . Pn _ P + Pn = 0 be the given
Pn +(: )pn-1 Pn- 2 Po = 0
1
differential equation of the first order and degree n > I.
the given differential equation is solvable for p if,
where P0, PP P2 , . .. , Pn are the functions of x andy.
[p- F1(X, y)] [P - Fi(x, y)] - [P - Fp.r)~

and : = p,(:r = p2 ...


=O
Take each factor equal to zero. we obtain 11
equations of first degree and first order.
[p - F 1 (x, y)] = 0, [p - Fi(,\:. y)] = 0. ...
or p = Fi(x, y), p = FN··_v). ···
or cp 1(x, y, c) = 0, cp,, (x; y. c) = 0. ...
The equation becomes, p n + p'Pn +-1p Hence the general solution is,
- 1 J/ n - 2
+ ... +p"P0 = 0 <l> 1(x, y, c) <l>i(.'C, y, c) <l>uCx, y , c) = 0
We have differe nt method for so lving th e
differential equation of first order but not of first degree.
Method I Equation solvable for P
Example-25. Solve: (dydx )
2
- 5 (dy
dx:
)+ 6 = 0
Method D : Equations solvable for y Solution: We are given that the differential equation

(ddxy)2- 5 (dydx )+ 6 = 0
Method III : Equation solvable for x
Method IV Lagrange's equation
Method V Claurit's equation or P2 - SP+ 6 = O
Method VI Reducible to Claurit's equation or (P - 2)(P - 3) = O
67 1 I
EQ UA TIO N
D1 Ff'E RE NT IAL I

2 O '
or P
nt ai ns ( : ) ,
implies P=2
Take the factor eq ual to ze ro w hi ch co
dy
- =2
or dx ~ ( x, p, : ) =O
or dy = 2d x or
ge t
y= 2 x+ c ov e eq ua tio n, w e
or O n solving, th e ab )=0
an d P -3 = o TJ (X, p, C
, p) an d
at e 'p 'fr om th e eq ua tio ns y = q> (x
dy Elim in
or - =3 TJ(x,p, C) .
dx en w rit e th e
is no t po ss ib le th
y= 3x + C If th e el im in at io n
or ric form
lu tio n is gi ve n b solution in paramet = Fi (P , C) .
So, Th e ge ne ra l so - C) =0 Y, x = F 1(P, C ) an d y
(y - 2x - C ) (y - 3x nt
where C is in te gr at
io n co ns ta

e: (ddxy)
2
2 +2
=0 dy
i.e.,

Example-27. So lv e:
xJ-x(: )-
4
(:
y =0
ua tio n
Ex am pl e- 26 . So lv X - -Jx ve n th at th e differential eq
dx . Solution: We are gi

J-x(: )-
.
on : W e ar e gi ve n th at th e d 1"f'C1 er en t1 al
So lu ti 4 y =0
equation, x (:
2
dy or x 4P2 -x P -y = O 4 2
dy ) + 2x dx - 3x 2 = 0 y= -x P + x P
( dx or
w.r. to 'x ', w e ge t
or P + 2x P - 3x2 =
2 O O n differentiating
)
2
P + 3x P - xP - 3x
2 = O 3p 2 + x4 (2 P dp
or dy = p -x dp + 4x dx
3x ) = O
P( P + 3x )- x( P +
or dx dx
(P + 3x ) (P -x ) =
O
or + (2 x4 P -x ) dp
P + 3x = O or P = (- P + 4x 3P 2) dx
or
dy
-+ 3 x = O 3P - I) dP =O
P (2 x P - I) + x( 2x
3
im pl ie s dx or dx

or dy + 3x dx = 0 3 =0
(2 x p - 1) [ 2 p +
X : ]
2 or
3x
or y+2 =Cl
=0
2p + xd p
P -x = O or dx
an d
dp
dy
- - x =O or X dx + 2p
=0
or dx
dp dx
x2
or - = - 2X-
y - 2- = C2 p
or
O n In te gr at in g, w
e ge t lo gp = -2 1o gx + lo g c
by,
So , Th e ge ne ra l so lu tio n is gi ve n or px 2= c
2
2 C
3x 2 - C1 ) (y - 2
x - C2 ) = 0 or p= -: ,-
x-
(y+
n co ns ta nt s. C2
ar e th e in te gr at io C
y = -x -2 + x4 -X 2
Where C l an d C2 Y or
r
ho d- II : E q u at io n s S ol va b le fo en Y = <P
X
M et so lv ab le fo r 'y ' th
n F (x, y, p) = O is
If the eq ua tio y= c2 _ C
or
X
(X ,p ). 'x ', w e ge t-
g w ith re sp ec t to ·n te .
O n di ff er en tia tin tra ry co ns ta nt of 1 gr a110n.
w he re C is an ar bi
p = 'V(x, p, : )
-=6~7.=2:.,_____ ___ ___ ___ ___ ___ ___ ___ ___
___ ___ ___:.,:_M::_ATHEMAT
Metho d-Ill: Equat ion Solva ble for x --_IC:s

lf the equatio nsf(x, y, p) = 0 is solvabl e for x, then --...


or P - F(P) = [xF'(P) + f'(P)] ~
x = c\)(y, p), on differen tiating with respect to 'y', we get
dx
dx = xF'(P) + J'(P)
dx
dy
= '11
(
y, p, dp
dy J or dP P- F(P)
Which On solving, we get, r,(y,p, C) = 0
or dx = xF'(P) f'(P)
+--.:. ._
where C is an arbitra ry constant, Eliminate 'p 'from the
dP P - F(P) P - F(P)
equations x = q,(y, p) and r,(v, p, C) . If the elimina tion is
not possibl e then write the solution in parame tric form or dx _F_'_(P_)- . x = ----'--f_'(_P.,;_)_
x = f/p, C), y = fi(p, C). dP P- F(P) P- F(P)
Examp le-28. Solve this differential equation,
which is linear equatio n with Integra ting factor
y= 2px + y2p3
Solutio n: We are given that the differential equatio n J F' (P) dP
y= 2px + y-p3 1.F. = e P- f(P)
The require d solution is,
or c\)(x,p, C) = 0
Now elimina te 'P' from the equations
Differe ntiatlng w.r. toy, we get y = xF(P) + f(P)
and c\)(x, P, C) = 0
dx = 1 _ y dp . 2yp 2 _ y2 _2P dp we get the solution, 'lf(x, y, C) = 0
dy 2p 2 p 2 dy 2 2 dy Example-29: Solve: y = apx + bp3.
Solution: We are given that the differential equation,
or _l_+ YP2 + dp(_Y _+ yp2 \ =0 y = apx + bp3
2p dy 2p2 ) On differen tiating with respect to 'x' we get

or
LPY + 2>1(p + y:) 0 =
dy
-
dx
= ap+axdp 2dp
-+3bp · -
dx dx

p= ap+(ax +3bp )dp


2
Take the factor equal to zero which have the term dp , i.e or -
dy dx

p(l - a)= (ax+ 3bp ) -dp


dp 1 2
p + y - = 0 or - dp + -1 dy =0
or
dy p y dx
On integrat ing, we get, dx _ ax+3bp 2
logp +logy _= log C
dp - p(l-a)
C
or py=C or p= -
y dx a x 3b
or -=----+-P
Substit uting this value of p in the given differen tial dp (1- a) p (1- a)
equatio n, we get dx a 3b
or -+---·x = --p
C+
y = (2x)y (C\
y2 y) or y2 = 2Cx + C3
dp (a - 1) p 1- a
which is linear differential equatio n with integrating factor.
where, C is an integration constant. . J - a- dp
Metho d (iv):La grange 's Equati on: The differen tial 1.F.= e
(a - l)p
equatio n of the form, y = xF(P) + f(P) is known as
Lagrange's equation. _!!__ logp
To solve this differential equation, differentiating . .=
IF e
a-I

this equation with respect to x, we get a


a -\
p = F(P) + xf'(P). dP + f'(P) . dP I.F. = p
dx dx Hence the solution is,
D1fl'£RENTIAL EQUATION 673
I
a I
Example-31: solve:
p~ x- - - P - p~
-f3b a-1d
I-a P+C
(x2 -a2 >(dy )2-2xy dy + y2 -b2 =0 \I
I
dx dx
- 3bp2 _ a_ I'
x- -+c a-J Solution: We are given that the differential equation I I

' ' · th · d"f


value of x m e given I ferential
2-3a
where C is an arbitrary constant of inte t·
p

gra ion. Put the


· equat·ion, we get
(x 2 - a 2{ :.J - ~~2xy + y2 -b
2
== 0
ii
I

_ ap ( -3bp2
y- _a
+ c pa-
2- 3a
1 +b 3
'P
l or
or
or
(x2 _ a2)p2 - 2xyP + y2 - b = 0
p2x2-a2p2- 2xyP+y2 - b =0
2
(y - Px)2 = a2P + b
2
2
2

or y-Px== ±-va-P +
r.? 2 b2
3 2a-l
3b
y = -a -
'P -
+ aC p a - I + bp3
2-3a or y == Px ± Ja P2 2
+h
2 . .. (i)

b 3 2a- l
Differentiating with respect to ' x' we get
2
y = _ P_+ac pa-l dy dP 1 2 dP
2-3a - =P+x-+-====•2a P· -
dx dx - 2 .Ja2P2 +b2 dx
which is required solution.
Method (v): Equation in Clairaut's form or Special case dP a2P dP
of Lagrange's equation: An equation of the form or P= P+x-±---:====
dx Ja2P2 +b2 dx
y = px + f(p) is known as clariaut's equation.
On differentiating the above equation w.r. to 'x'. We get 2
or
+ a P ldP_ 0
P= p+xdP +f'(P/P (x .Ja2 p 2 + b2 dx -
dx dx
dP
I dP dP or =0 or P= C
-
[x + f (P)]- = 0 or - = 0 clx
dx dx
Putting the value of P in equation (i), we get
On integrating, we fet, P = C
Then, y = Cx + f(c) y == Cx ± .Ja c + b
2 2 2

Examplc-30: Solve: cos (Px - y) = P


Solution: We are given that the differential equation, where C is an arbitrary constant of integration.
cos (Px -y) = P Example-32: Solve
or Px -y = cos- '(P) ory (x + a)P2 + (x - y)P - y = 0
== Px - cos- 1(P) Solution: We are given that the differential equation,
Differentiating w.r. to 'x', we get (x + a)P2 + (x - y)P - y == 0
dy dP 1 _dP or xP2 + aP2 + Px - Py - y = 0
dx = p + X dx + fi- p2 dx or Px(l +P)+aP2 ==y(P+ \)
2
aP
1 dP
dP or y = Px+-- ... (i)
P= P + x - + ~2 dx \+P
or dx ✓1-P
On differentiating with respect to 'x' we get

or (I+ pl: = O or
dP
dx - O dy
- =
dx
P+x-+
d.x
~ dP
dx
(I + P)2
, dP
dP (I + P) LaP - - oJ• - _
clx

or p=C . . I
. th given d1fferent1a
Putting the value of C 10 e p == p + x dP + (I + P)2aP - aP dP
2

equation, we get _1 C or 2
(1 + P )
y = ex-cos . dx dx
. .
where C 1s an arbitrary constan °
t f integration.
""
MATt-tet.i,,r,es
674
which is clairaut's form. The required soluti on~
*C)
2
I+aP ] dP dP
or
[ x + (I + P )2 dx = O or -dx =O
V = Cu + kf (
or P= C
On putting the value of ' P' in equation (i), we get
ac 2
yP = Cx
0
+kf(f C)
y = Cx + - -
I+C Example-33: Solve: y = 2px + y2p 3
where, C is an arbitrary constant of integration. Solution: We are given that the differential equation
Method (vi): Equation Reducible to Clairaut's form: y= 2px+yp3 '
Some equations can be reduced to Clairauts form by using It is reducible to clairaut's equation by putting,
the suitable substitution.
Workin g Rules for Solving These Type of f
a.
= 2or P=2a.
Equatio ns
Rule I: The given differential equation is or a.= l, p= 2
Let u=x, v= y2
/3 ·+---f
y = -px k (px I- Cl P- 1)
y
a y (/3 -1) du dv dy
put u = x n, v = yP dx = 1, dx = 2y- dx =2yp
du . dv
Then - = axa-1 Hence -du = 2yp or P = 2yp
dx
dv = pyP-1 dy
or
p
dx dx p=-
2y
dv /3 y P-I Putting the value of p in the given differential
Hence, -=---P
du a. xa - 1 equation, we get
dy dv 2P 2 P
3
dx =preplac e - =P y = - x+y -
du 2y sy3
/3 yP-1 p p3
P= - - - p
a xn-1 y = -x+-
y Sy
a xa -1 p p3
p= 13 yP-1 p y =· - x + -
y Sy
On putting the value of 'p ' in the given equation, p3
we get or y2 = Px + -
S
p a. xa - 1 k
y= - · - - P -x + - f -
xa -1
-P-
xl -n ) [a On solving the above equation, we get
dy
a p yP-1 yP-1 p yP-1 yP-1 - =CorP =C
dx
y =x- P +
yP-1
yP-1
0
k- f -P
p (a ) Hence, the required solution is,
c3
y2 = Cx + -
S
0
y P= x P+kf(f p) Rule II: The given differenti al equation is
(a - bp)eby = f(peby - ax)
V = Pu + kf ( *p) or
On putting u = eax and v = e6Y
-du = aeax and -dv -- beb.v . P
dx
dx
DIFFERENTIAL EQUATION 675
3 2
dv _ beby p Example-34: Solve: e3x(p - 1) + p e Y = O .
du--;;;a;- Solution: We are given that the differential equatwn,
b e3x(p - l) + p3e2y = 0
p = -eby-ax .p or (I - p)e3.Y = p3e2Y
or a
( 1 _ P) =P3 e2Y - 3x
p a
= -Peax-by (l _ p) = p3e-3x. e2Y
or b (1 - p)eY = (pe-xey)3 .
On comparing the above equation with the equation
. On putting the value of 'p' in the g·iven d.f"" .
1 1erent1al
equatton, we get (a - bp)eby =f(peby-ax)
we get, a=t , b=1
(a- b ~ Peax-by }by = f ( ~ Peax-by . eby) Let u = eX and v = eY
dv
du= eX = eY . p
or (a- aPeax-by)eby = 1(~p) or dv dx

dv eY · p
or -=--
du ex
or
or p = eX-Yp
On putting the value of p in the given differential
or equation we get
(1 _ eX-YP)e3x = e3x-3yp3e2y

or v= uP+;f(~p) The required solution is given by


eY= CeX + C 3
which is required clairaut's form.

SINGULAR SOLUTION~

Singular solutions are noting but the envelop of the family and its general solution is,F(x, y, C) = O
of the general solution or the equation of the envelope is which represents one ·parameter family of curves.
the singular solution of the differential equation. . A solution does not contain any arbitrary constant
Let the given differential equation is and is not a particular case of general solution by assigning
.f{x,y, p) = 0 any value to the parameter C, is called a singular solution
of the given defferential equation.

Remark: A curve is said to be envelop


·~ -,,.,~
:tr eyeJx)~?int'oft~i,~u.r,yd :ti>u6h~s -~ni -~f!the~olutirin to the family of the solution.
,:_ ·-..- ,';,.:;.·-.·j/ ;·_..-;;/;,- _;.-~_•. -. .,J,.~-.:.->>"!,~,,;i~-~"'·· . ' --~- .... ·, .• .,., .

Geometrical Significance of Singular Solution p R


Let the family of curves possesses an envelop PQ and A(x, y) be any point on the envelope.
e family touches it. Further RS is a solution curve of the differential
Some curve RS o f th A( ) R'S' ·
equation and the curves RS and R'S' have same values of~• y, Pat x, Y , 1s also a solution
of th . Th. t· ·s di·ffierent from the solution curves represented by the general ~s
e equation. 1s so1u 10n 1
solution and is known as singular solution.
ily is obtained by eliminating parameter C between the R'
N ow, the envelope o f the f:am
equations
aJ = o
F(x, y, c) = 0 and ac Q

Which is the required singular solution.


MATHEMA->-
676 "' ics

Remarks:
I. Singular solution does not contain any arbitrary constant.
2. Singular solution can not find by giving the particular values to the general solution.

Workin g Rule to Find the Singula r solution Example -35. Find the singular solution of the
p-discriminant: Let f (x, y, p) = 0 be a given equation,
p 3 - 4xyp + 8y2 = 0
differenti al equation. p-discrim inant is obtained by
eleminating p between the equations, Solution: We are given that the equation,
f(x , y,p) = p3 - 4xyp + 8y2 == 0
f(x, y, p) = 0 and
aJ
-=O On differenti ating the above equation w.r. top, We ·
aP get
The required general form is,
-
a1 =3p2 -4xy=0
Llp -ET2C= 0
ap
where LlP ➔ p-discrim inant
E ➔ Envelope
or 3p2 = 4xy or p 2 = -4 xy
T ➔ Tac locus 3
C ➔ Cusp locus.
. C-Disc~im_in~nt: Let.f(x, y, c) = 0 be the general
solution. C-D1scnm mant is obtained by Eliminati ng C
between the equations
or
p~ ✓~xy or p3~

On putting the above values in the given equatio


(~xy r
~~ ~
f (x, y, C) and aJ = o
The required solution is, Llc _ EN 2c 3
ac
=0 ✓~xy(~xy-4x+:-sr ~
where, ilc ➔ C discrimin ant£ ➔ Envelope
N ➔ Node locus ~( -~xy ) ~-sy2
C ➔ Cusp locus.
Special Cases:
.
I. If the family is quardaratic in C l.e., or ~(1)~r
f(x , y , p) =Ap2 + Bp + C = 0 Squaring both sides, we get

then a1 4 x2y2
ap = 0 or 2AP + B = o -xy -- =y4
3 9

or -B or 3 27y4
P=-
2A x y3 = 4 or y3(4x3 - 27y) == 0
or 3
On putting the value ofp in the given . Y (27y - 4x3 ) == o
get- equation, we . .
The general solution of the g·
1ven equation 1s,
2 Y == C (C-x)2 .. .(i)
BJ+B(-B)
A( 4A2 2A +C =O
-ac ==o
dy
or
or B2 B2 (C-x)2 + 2C(C-x ) == o
---+ c -- 0 or B2 - 4AC = 0
or
_ \C-x)(3 C-x)== O
4A 2A
Ehmmati ng 'C' fi ... (Z)
2. If the given e . . get rom the equation s (i) and (ii) we
quation m Clairaute 's form:
h · y = px + f(p) d C-x==O
w ere, pis p-discrim inant and C is _an y == Cx +/(c) implies
are same only when they h C-d1scriminant which C==x an d 3C -x = 0
ave same envelope.
or X
C== -
3
OiFFERENTIAL EQUATION
677
rutting the values of C in the general .
so1ut1on, we get The common factor in p-discriminant a nd C-
= 0 and discriminant both is,
Y y = _4x3
27 y(27y - 4x3) = 0
Hence, y = O or 27y- 4x3 = 0. These two sa~isfying
C-discrim inant is, Y( Y - :; ) = o y(27y- 4x') = 0 the original solution and hence these are the smgular
solution.

i CHECK POINT EXERCISE_ _


14 1
Solve the following differential equations :
dy eY 1
dy
1. dx + Y tan x == x2 cos x 6. dx == x1 - ~
7. .xy" = y' + y'3
dy
2. x dx - Y = 2x2 cosec 2x 8. dy - tan y = (1 + x) ex secy
dx l+x
3. xdy - ydx - 2x3dx == O 9. 2.xydx+(l-x2 +2y)dy==0
4. (x + 2y3) dy = ydx
10. 2x2 dy = -2xy + x2 + 3y2 given that y 1(x) == x is a
dx
S. dy + y d<j> == <j>(x) d<j>
solution.
dx dx dx
11. y lny dx + (x - lny) dy = 0

APPLICATIONS OF ORDINARY DIFFERENTIAEO:~QlJATi€>1',f'S:1' 1( ''.

In this section we discuss the mechanical applications of Solution: We are given that the equation
ordinary differential equation. The applications of ordinary dN
differential equations as follows: - - kN0 N = -kN2
dt
I . Growth of substance Substitute z = N- 1, the equation becomes
2. Newton's law of cooling dz
3. Decay of substance -+kN0 z =k
dt
4. Motion in resisting medium
which is linear equation with integrating factor,
5. Dilution
6. Escape velocity I.F. = ef kNo dr = ekNot
l. Growth of Substance: The rate growth of
The general solution is,
substance is dir~ctly proportional to the
substance present or
z ekNot = JkekNot + C
dN aN ekNot
dt or zekNot = --+ C
No
dN == kN
or
dt or -1 = - 1 + C e-kNot
. d k is positive constant. N N0
where, N is the substance at time tan ce satisfies the At
Example-36: The growth of a subS tan t = 0 N= N(O)
logistic equation 1 1
then C= - - =- -
dN == kN(Nn-NJ, wherekandNo N(O ) N 0
Substitute the value of C in the equation, we get
dt tes the substance at
are positive constants and N(t) dezo wth ofsubstance
time t. ffN(O) < N 0 then show that t e gro
can never exceed the number No·
~ = :, +(N;O) - :, ),·kNo
~ 678
MATH EMA Tlcs
3. Decay of a Substance: The rate of
decay f
substance is directly proportional to a
substance pres:n:
we have,
'
dN
- - ex: N
dt
dN
or - - = kN
dt
where k is positive constant and N(t) is amo
Since, N(O) < N 0 then ( ~ -
N(O)
l)e-kN° 1
is positive at time t.
unt of substance

for all t and tends to O as t ➔ 00 • Hen ce, Example-38: In a radi oac tive dec ay,
from the solution, initially 50 mg of
N < N 0 for all t and N ➔ N as t ➔ 00 the mat eria l is pre sen t and afte r two
ther efore N nev er hou rs, the material
exc ed N 0 . 0 ,
hav e lost I 0% of its orig inal mass. Fin
d the mas s at any
time t and the hal f life of the mat eria l.
2. New ton 's Law of Cooling: The rate
of change Solution: Her e,
of tem pera ture of a body is directly
proportional to the
tem per atu re diff ere nce bet wee n N= 50 mg
the bod y and its Let Nbe the amo unt of the material pres
surr oun ding medium . ent at time
t, then
or dT
dt ex: -{T -T)
I
dN
- =-k N
dt
or dT On solving the abo ve equation, we get
dt = - k(T - T')
N= Ce-kt
whe re T den otes the temperature of the at t = 0, N = 50 then
bod y and T denotes
the tem per atur e of the surr oun ding
med ium and k is N= 50 e-kt
pos itiv e con stan t and at t= 2, N= 45 then
T > T' 45 = 5oe-2k
Exa mp le-3 7: A body is hea ted to I 00°
C and placed in air 1 9
at 20° C. Afte r one hou r its tem pera ture implies k= --l n-= 0.0 53
is 60°C. How muc h
add itio nal time is requ ired for it to coo 2 10
l to 30°C ? mass at time t,
Sol utio n: We are give n that the equatio N = 50 e-.053 r
n
dT
- + k (T- T) =0 If !L be the half life, then
dt 2
Her e T= 100°C t
at t= _1_ N = 25
and T' = 20°C 2'
Now, dT --0.53~
- + k (T - 20) = 0 implies 25 = soe 2
dt
whi ch is line ar equation. The required
solution is, implies !!_ = - 1-ln 2
T= 20 + Ce- k1 2 0.053
At t = 0, T = 100°C implies C = 80 or ti
-2 = 13 hou rs
At t = 1, T= 60° C implies k = In 2
.
Hen ce, 4. Motion in a Res istin g Medium: A bod
T = 20 + 80 e- 1 In 2 y is falling
~nder gravity 'g' in a resi stin g med ium
whose resistan~e
1s proportional to the velocity v. Acc
For T= 30°C e-11n 2 = -1 seco nd law of motion,
ordi ng to Newton 5
, 8
t In 2 = 3 In 2 dv dv
t = 3 hou rs
mdt = mg - mkv = mv dx
Hen ce, additional time is 3 hours. dv
or dv
cit = g - kv = v dx
Ij
[)tffERENTIAL EQUATION ,6,7 9 I

The ~esistance is directly proportional to th . . d from the Earth's


lif the velocity, e square If the body is proJecte~ upwar b k this
surface with a velocity so that it may not return ac '
dv f et
-dt = g-kv2- dv velocity is called escape velocity. .
- v- If v and x are the velocity and the distance at im . ,
.. dx · acceleration due to gravity
• and g is
where k an d g are positive constants and x is the ct·istance Vbe the escape velocity
- , ,
at time 1 · and R is radius of Earth, then
Example-39: A particle falls down from '~e· st zn . h .
. • ..
whose resistance zs preposztzonal to the
t e azr , dv = _J::._ v(oo) = 0
. d h I . square of the v- 2'
velocity. F zn t e ve_oczt;: as a function ofx. dx X

Solution: For velocity, distance relation µ


dv
' v(R) = V, g = R2
v dx = g-kv2
On Integrating, we get
2kvdv
or =-2 kdx v2 = 2µ + C
v2 _ .?__ X

k v(oo) =0 implies C =0
On Integrating, we get 2gR
2

v2 =- or
1n ( V
2
- ! )= - 2kx + lnC 1
then,
Further, v(.Rj = V
X X

implies V 2 = 2gR2 /R
So v2 = .?__ + C e-2kx
k I V= .J2gR
For Earth, g = .0098 krn/sec2 and R = 6400 km,
At t = 0, x = 0, v = 0 implies C = _.?__ hence escape velocity, V = 11.2 km.
I k
Trajectories: A curve which intersects every
Hence, v2 = ..[ (1- e-2kx) member of a given family of curves in accordance with a
k given Rule is called a trajectory of that family. There are
5. Dillution: A tank contains V liters of a salt several kind of trajectories.
solution that contains a kg of salt and another salt solution 1. Orthogonal Trajectories: In a trajectory if each
of b kg of salt per liter is poured into the tank at the rate of member of a family of curves intersects each member of
r liters per minute. another family of curves at right angles at every point of
If Q denotes the salt in the tank at any time t, then intersection, Then one family is called a family of
orthogonal trajectories of the another.
its rate of increase is dQ which is equal to the rate of (a) Cartesian Coordinate Syst'em: Let us consider
dt ' a One parameter family of curves
increase due to flow in over flow out. F(x,y, a)= 0
Now salt enters the tank at the rate of b.r. kg per
where a is parameter. Now, differentiating with respect to
minute, the 'volume of the solution at time t is v _+ rt - ~t x, we get
and the amount of salt is Q, the rate of dramage is

s -
_ _Q::::c!.. - kg per minute. 'V(x, y, : ; a)=0
v+ (r - s)t
Eliminating the parameter 'a' from above two
dQ = br- Qs families, we get

dQ + S
dt

Q=br
v+(r- s )t
$( y,:) =
X, Q

or dt v + (r - s) t Now at any point (x, y ) on any c urve of the family


at t = o, Q = a and Q(O) = a which is linear equation. We
can so1ve it.
·
h vitational force
y F (x, y,
.
a) = 0 or $ (x,y, ddxy) = 0 , T he curve has the
6. Escape Velocity: In space, t e gra th Earth's
Varies inversely as the square of the distance to e dy d h .
slope dx an t e orthogonal t rajectory thorough the same
centre.
680
7 I

MATHEMAlJcs
Example-41: Find the ortho gona l traje ctori es
point has its slope a~ - : ; , so that the product offamily
of two of curv es y = ax2, wher e a is a para mete r.
Solution: We are given that the family of curves
is
slopes is -1. Therefore , 1'f x, y, -dy charact enze
. a curve o f y=a x2
dx wher e 'a' is a para mete r
the family differentiating, with respe ct to x, we get

<1>(x, y,: )= 0
dy
- =2a x
dx
dx .
Then x, y, - - char acte rize
dy
an orth ogon al usin gy = ax2 , we get dy = 2(.1.2...). x
trajectory through the same point. Thus , the diffe dx x
rential
equation of the family of orthogonal trajectories
is given
. dy or equivalently
by rep1acmg - by - -dx .m
dx dy
Repl acing dy by - dx , the differential equation
Q) X,
( dy) _0.
y, cfx - dx dy
of the required orthogonal trajectories is given
by
by solvi ng it, we get the family of orthogonal traje
ctories
of the family F(x, y, a)= 0 asf( x, y, A)= 0, wher
e A is a
- dx = 22'..
parameter. dy X
or equivalently xdx + 2ydy =0
Exam ple-40: Find the orth ogon al traje ctori es
of the on integrating, we get
diffe renti al equa tion offami ly of curv es
x2

(:J = ;·
Solution: We are given that the differential equa
tion of
-+ y2 ='.A.2

x2
2
y2
or equivalently, 11,2 + A2 .= l
the family of curve 2
which is the required ortho gona l trajectories, wher
a e Ais a
parameter.
X
(b) Polar Coordinate System: Let us consider
a
Replacing dy = - dx , the differential equation one parameter family of curv e in pola r coordinate
of as
dx dy F(r, 0, a)= 0
required orthogonal trajectories is
where a is parameter. Differentiating with respe
ct to 0. we
a get

or eq uival ently
X
~(r,e,:;.a)=o
On integrating, we get
dy= ±(:: )dx
Eliminating parameter 'a' from above families. we
get differential equation of the family
F(r, 0, a) = 0

as <1>(r, 0, dr) = 0
d0 ..
or equiv alently Now at any point (r, 0) of any curve of this faJlll)1

3✓a( y + A) = ± 2x 312
. cir
the slope 1s de .The differential equation of the orthog0 11nl
or equivalently
9a(y + 'A.)2 = 4x 3 ·
traJe ctories is give n by replacing -dr by - : d0 in
which is the required orthogonal trajectories, wher
r -
e A is a d0 dr
parameter.
<p(r,6, dr) =0
d0
DtFFERENTIA L EQUATION /l
681
by solving it, we get the equat·
th
orthogonal trajectori es of the famiJ~on of e family of where a is a paramete,:
F(x, y, a)= o Solution: We are given that
as f (r, e, A.)= O r' ccs n e = an
where a is parameter. Taking logarithm of both sides, we
where A is a parameter .
get
Example-42: Find the equation oifth
. e system of thogonal n log r + log cos n 0 = log an
trajectories of the parabola s or differentiating with respect to 0, we get

r= - - - -
2a
( ~ )( :; )- n tan n e = 0
1 + cos e
where a is parameter.

u~)(:;
or equivalently
Solution: We are given that
2a )-tan ne = 0
r= - - - -
1 + cos e · 1
where a is paramete r Replacing -dr ) by -r2 -d8 , we get the d"ffi
I erent1a.
( d8 dr
taking logarithm on both sides. we get of the required orthogonal trajectories is
log r = log 2a - log ( I + cos 8)
Different iating with respect to e, we get (; }-r 2
) (~~)-ta n ne =
0
( ~) ( :; ) - 1 ;i;o~ e
or equivalently (; )dr + cot n 0 d8 = 0
Replacing (
d0
rlr) -r
by
dr
2
dS we get, the differential
On integrating, we get
equation of the required orthogon al trajectorie s
is log r + ( ~ }og sin n e
2 sin (0/2) cos (8.12) log A-,
2 or equivalently
2cos
n log r + log sin n e = n log "A,
(~)

dr 0 or equivalently
- = -cot-d0 rn sin n e = ')._n
or equivalen tly r 2
.where ')., is a parameter.
On integratin g, we get 2. Self orthogonal Trajectories: If each member
log r = -2 log sin(0/2) + log A
of a given family of curves intersects all other members
').., orthorogonally, then the given family of curves is known
r= as self orthogonal.
or equivalen tly sin 2 (0/2)
Note that if the differential equation of the family
1- cos e of curves is .identical with the differenti al equation of its
or equivalen tly sin 2 (0/2) = 2 orthogona l trajectories, then such a family of curves must
z trajectories of be self orthogonal.
Example-43: Find the orthogo: a
,-n cos t10 = a
CHECK POINT EXERCISE - 14.2 . 3. A Certain radioactive material is known to decay at a
. d bled in 50 years, tn rate proportional to the amount present. lfafter one hour,
1. If the populatio n of a country tis d ogu1·ven that the rate of 10% of the material is decayed. Find the half life of the
·11 · b treb e , ber of inhab1tan· ts.?
how many years w1 it e material.
increase is proportion al to the num e of 1oooc. Then 4. Find the orthogonal trajectories of the following families
temperatur ture
2. A copper ball is heate d to a . tained at a tempera of curves.
it is placed in water which is main the temperature· of the (i) Family of straight lines
3 · utes h the n + 2y = a, a being parameter .
of 30°C. At the end of mt~ h time at whtc
700c Find t e
ball is I educed to . · d ed to 31 °C.
temperatu re of the ball ,s re uc
;~ ;; :~ ;: ~~ ~~ ~~ ~- -- M _A T H _~
EMAl'rc:s

2 -- -- -- -- -- -- -- -- -- --
~~ -- ;- ogonal trajectories
s. Find orth a being parameter. of
~8 (i) r.= e0 ",
(ii) Family of semicubical parabo
las parameter.
(ii) r(1 + cos 0) = 2a, a being
ay = i3, a being parameter. Show that the following familie
s of curves are Self
6.
(iii) Family of concentric circles orthogonal
. x2 + y2 = a2, a being parameter. (a) y2 = 4a( x + a), a being par
ameter.
2
(iv) Family of curves 2
= I , I being parameter.
r. + Y
x2 - y2 = ax, a being paramete ( b) x
2 2
+ A)
(a + A) (b

IE NT S
TIA L EQ UA TIO N W IT H CO NS TA ~T CO EF FIC
LINEAR DIFFEREN
n
endent variable and its Co mp lem en tar y Fu nc tio
An equation involving the dep g hom oge nou s equation is
first degree and are not The solution of correspondin
.
differential coefficients only in the called complimentary funct10n.
ar differential equation
multiplied together is known as line or [f{D)Jy = 0
coefficients of this linear
with constant coefficients. The Auxilliary equation is giv en by,
t.
differential equations are constan /('A,)= Ohas roots -~ \, A2, ... A
form ,
The differential equation of the auxilliruy eq~tion
Case I: When all the roots of the
dny d" - ly
,,y =F (x) are real and distinct :
ao d.x" +a1 d.xn-l + . .... . +a l and distinct roots
Let Ai, A2, ... \ be the n rea
1 )y= F(x )
(a0 IY1 + a 1 D - + .... .. + a,,
11
or then,
equation with constant
is known as linear differential ,, e"·•x
'# 0, a/s are con sta nt C.F. = CI eAiX +C 2 eA2X +.: .+c
coefficient of ord er n and a 0
ati on has m equal
Case II: When aux illi ary equ
.. An
and Dis.!!:.....
d.x roots. i. e A1 = A2 = ... Am, A,m+ I'
n is given by,
linear differential Then the complimentery functio 2
The general solution of the C.F. =(C 1 + Cr + C3x + ...
+ Cmx"' - 1)
t is
equation with constant coefficien '
y = C.F. + P.I. e"·mx + .$m+I eAm+lx + .. C n eAnx
ction and P.I. is particular ~a se III : When the roots are
complex conjugate
where C.F. is complimentary fun ..
integral defined as follows: or a+ zP and a - ip then,

C.F. = eax[C1 co sP x+ C,-.sin


~x]
, ...· .•
·. / . > ....·
n' 26m ·'1 .. /\ ,; ·. . . . . {.
i .
the differential equation are real the. . 'J p ,ex roots are always exists. m conJugate but if they
Remark: lf the coefficients of ut them . .
are not real then we can not say abo
If roots are a + jp and a - Jp
, Then
C.F. = eax[C1cos h/p x+ C2 sin hJfj x]
or .
c
cie<a+.ffi>x +' ;i.e<a. - .ffi> x-
,/I '

nta ry Jun ctio n o• the .


E_xampl~-44: Fin d the com ple me 'l Here , the roots are com p 1ex cotlJ ugate, then
dif.ferentzal equation, C.F. =
(D2 + D + I) y == O
Solution: We are given that the
(D2 + D + 1) y = O
differentia l equation
' , -1/2, ( Cl COS: X +C, sin: x)
c
where C 1 and 2 are arbitrary constant. .
The auxilliary equation is
Example-45: Sol ve
11.2 + 11. + I == O '
(D4 + m4 ) y = 0 .
or 11, == - l ± i✓3 •
Sol uti on· We are giv dif fer ent ial equation,
· en tha t the
2 (D4 + m4 ) y = 0
D1FfERENTIAL EQUATION
683
---~T;he: aa:ux~i~l l~ia~ry~e ~q;u~at~i ~on~is~-- ----------- ----------- ----~~ ~
(A4+m4)=0 ' y = (C1 + CzX)e-x
(A2 + m2)2 _ 2A2m2 == 0 where C 1, C2 are arbitrary constant.
or
(A2 + m2)2 _ (✓2 Am)2 == 0 Particular- I ntegr-al
or
or (A2 + m2 - ✓2A m) (A2 + m2 + ✓2 Am)== O The expression y = _l_ is defined as the function of x.
f(D) _ . .
A2 + ✓2m A+ m2 == 0 The particular integral of the above expression 1s given
If
by,
A. == -✓2m ± ✓2m2 - 4m2 l
Then P.I. = f (D) X.
2

or A. == ..!!!:_ ± i ..!!!:_ General Method to Find the Par-ticu lar-


✓2 ✓2
lntegr-al
and if A2 - ✓2 mA. + m 2 == 0
Theorem-I: IfXis a function ofx then

A. = - ..!!!:_ + i ..!!!:_ I X f
= ea.x Xe-a.x dx
✓2- - ✓2 (D-a)
So, The required solution is - 1 X
' Proof: Let y- (D-a)

y= e
Jix(C 1 cos ✓
m
2 x + C2 sin
.
J2, x ) On operating by (D - a), we get
(D-a)y=X

)Y =X
+ehx(cl cos J2_ x+ C2 sin h x) or ( ; -a

or dy -a.y =X
where, C 1 and C 2 are arbitrary constant. dx
Example-46: Solve: (D 2 + 2D + J)y = O which is linear differential equation with integrating factor
Solution: We are given that the differential equation,
I.F. = e
-f cube = e-a.x
(D2 +2D+ l)y=O
Auxilliary equation is, The required solution,y e-a.x = f e -ax X dx
. A.2 + 2A + I = O
or (A.+ 1)2 = 0 y = ea.x f Xe-ax dx
A.= -I, -1.
Similarly, we can prove,
The auxilliary equation has repeated roots. Hence,
the required solution is, 1 X = e-ar f Xea'"d~
(D+a)

Remark· - -- - :· = . 't\ •.. .· · .-· ,_. .·•.- · ·


-:0,. • ·

_ tan· t of.m·
' shaJI •never a dd a cons
"we .• · -.- .. · •. ·: _ _.ation is·p_
t
tegra•._io
_ ri:after ·
1ntegr _· ··
· :\e__- :rformaj - method to finding P.l.
·· with any

Theorem-2: If n is a positive integer then, = - - - -1 ea.r · X


II (D-a)"-
] X ax
- - - - ea.x == - e l l .
(D-a.)'1 u.! = (D - a)" - 2 D - a x/x,
Proof: Taking L.H.S.,
I a.,
1 ea.x
1 I eax
)" _ 2 e Jxdx
== (D-a.t-' (D-a)
(D-at [by theorem ( I)]
1 .,
I eaxJ eax . e-ar dx = ---- - e <Y-r
·x-
-
(D-a.)'1 - 1 (D- a.) 11 - 2 [2
[using theorem (1)]
MATHE1.1
6,~~:__--------------~----Th;-;;;~~~~~:----------....:__"l1c
~ The required solution is, ~
2
1

ea.x x y = C.F. + P.I.


= (D-a.)"-3. (D-a.) 2
C
y = (C, - C2 x) e +
x
3e
3x
+ l [xe 3x - x 2 ex]
) 8
I m X
- - - - - : ; -3e ·- where, c, and C 2 are arbitrary constants.
- (D-a.)"- Ll Special cases
[again using theorem (I)]
Continuing this process, we finaly obtain, I e ax = -I- eax , I"ff(a)cf. O
Case I: - -
f(D) f(a) .
1 x" ax
- - - e ax = - - - - - e
(D-a.)" (D-a.)"-" U! l ax x" eax
Case II: (D _ a)" <p(D) · e = [!! q>(a) If $(a);t:o
II
X ax
=-e
U! I . I
Theorem 3: (i) 2 sm ax= sin ax
II
f(D ) /(-a 2 )
ru: X -ax
Similarly, ---e =- e Iff(-a2 ) -::i:- 0
(D+a)" ~
Example-47: Solve: (D3 - 5D2 + 7D - 3) I I
(ii) - - 2
cos ax = 2
cos ax
y = e2x cos hx f(D ) f(-a )
Solution: We are given that the differential equation,
Iff(-a 2 ) -::i:- 0
(D3- 5D2 + 7D-3)y = e2Y cos hx
The auxiliary equation is, Theorem-4:
m 3 - 5m 2 + 7m - 3 = 0 I X
- - - sin a x = - - cos ax
m - m - 4m 2 + 4m + 3m - 3 = 0
3 2 D2 + a2 2a
m2(m - I)- 4m(m - l) + 3(m - I)= 0
(m - I)(m 2 - 4m + 3) = 0 1
Proof: Taking imaginary part of 2
/ax
(m - I)(m -3)(m - I)= 0 D +a2
two roots are repeated. Hence,
I .
C.F. = (C 1 + C2x) e>: + C3 e3x = Imaginary part e'ax
2 2
D +a
2
Now, P.I. (D 3 +SD}+ D _ ) e cos hx • I
part e·'ax
x
7 3 = I magmary 2
? eOr·
(D+ia) + a-

PI = - - - I - - e2x (ex+e-x ) x .
. . (D - 1)2 (D - 3) 2
= Imaginary part - - e'ax
2(ia)

I I = Imaginary part of_.!_ x[cos ax+ i sin ax]


PI = - - -2- - - (e3x +ex)
. . 2 (D - 1) (D - 3) 2
-x
= -cos ax
P.l. = I- [ - - - - I- - e 3 x + I ex ] 2a
2 (D-1)2 (D-3) (D-I/ (D-3) 1 . X
Hence, 2 cos ax
2 sm ax =--
2
D +a 2a
I[ I 3x 1 x ] Similarly we can prove that,
= 2 (3 - 1)2 xe + I - 3 [1 ex
1
3 2
2 2 cos ax = ~ sin ax
D +a 2a
= _!_ [ xe + x ex ]
2 4 4 Example-48: Find the PI. of the differential equation,
(D2 -2D + l)y = cos 3x
I
P.I. = 8 [xe3x - x2ex] Solution: We are given that the differential equation,
(D2 - 2D + I )y = cos 3x
Dll'FERENTIAL EQUATION
685
we need to find the P.l.,P.1. = 1

=
(D2 _ 2 D

I
+ l) cos 3x
= -¼[ X: _X: + !~ X]
_ 32 _
2 D + l cos 3x
1
P. I = - /s [x x: + x]
3
-
2
:
= -_--_--D- cos 3x Example-SO: Solve,
8 2
(IY - 3D + 2)y = sin re-X) _ _
1 1 Solution: We are given that the d1fferent1al equat10n,
= - 2 D+4cos3x (D2 - 3D + 2)y = sin (e- x)
The auxilliary equation is,
_ 1 1 (A2 - 3tc + 2) = O
- - - D2 cos 3x or (A - 2)(A - I)= 0
2 +16
or A=-
1, 2 2x
1 1 So, C.F - C 1~ + C 2e •
cos 3x
2 - 32 +16 1 . ( -x)
N ow, P-1 = -2- - - - sm e
D -3D+2
1
= [-3sin3x-4cos3x ]
1
50 = . ( - f.)
~n e
. 1 (D-2)(D-1)
P .1.= - - [3sin 3x + 4 cos 3x]
50 1 - [ - - -1S l D
= --
(D-2) (D-1)
. (e -x)]

Eva Iuat1on o f --
1 xn , where n
f(D)
is a Positive 1 e xs e-x sm
= -- -
e -x dx
Integer D-2
If Xis form of x11 (polynomial), we use (Let e-x = t Implies - e -xdx = dt )

n(n -1) 1 exf smt


= -- . dt
2
(1 +xt= l+nx+ 1
,, x + ... D-2
tl
1 e cos - x
= -- X (
e )
1 2 D-2
Example-49: Find PL (x + 1)
D 3 -D 2 -6D
Solution: We need to find the P.I of the given differential · = e· - - 1--cos (e - x)
X"

(D +1- 2)
equation. So
= e·' --cos
1 .
(e- ·')
PI= 1 (x2 + 1) D-1
. D 3 - D 2 - 6D
= e-' -e-' Je-x cos e -x dx

=- 6~[1+(~ - )f ~ 2 (x'+l)
(Let e-x = t implies - e-x dx = dt)
= -e2 x Jcos t dt

=- 6~[1-[( ~ -~2)]ex'+ 1) +[( ~ - ~2)]cx2 +1)- ]


So, P. I = -e- 2-.: sin e-x
The required solution is, y = C.F + P.l
Y = c .e-' + C,,e 2 -.: - e-2 , sin (e-X)
where, C 1 and C 2 are arbitrary constants.
= __l_[x2 + 1-.!..(2x - 2) + 316 (2)] 1 ax cc,
6D 6 Theorem-S: - - - e V =e - - --V
1 (D) f(D+a)
Example-SI: Solve:(D 2 - 2D + 4)y = ~ cos x
=-
6
~[x 2
+1-½ x + ~:] Solution: We are given that the differentia l equation,
(D2 - 2D + 4 )y = e-' cos x

= - 6~[x2 -1+ ~~] The auxilliary equation is,


~86
-~~-------------------=-=~=:-:--~
A.2_ 2A. + 4 = O or
= eX
- I +3
ex COS X
'
2 COS X
MATHE,,i~l

So. C.F. = e' IC, ws fix+ C2 sin fi, xi = ---


2
_ I t!" cos X The required solution of the given differential
and P.1. - (D 2 -2D +4)
equations is,
- (' I I .''OS
. .X y = C.F. + P.I.
- (/J +l}2-2(D+ l)+4
. r;; ex cos x
r I = exrc, cos ✓3 X + C2 sin -v3x] + 2
= e - 2 -cosx
D +3

HOMOGENEOUS LINEAR EQUATIO N

Equation reducible to constant coefficien ts (D2 -D - 3D 1 - 5) y = e2z sin z


1
or (Cauchy-Euler's equation) (Df-4 D 1 - 5)y_= e~ sin z
The auxilliary equation 1s,
A linear differential equation of the fonn
Df-4D1 -5 =O
or (D 1 -5)(D1 + l)=O or D 1 =-1,5
d,1-l d
d Y/1
Y + .... .. a,,_ x-+a,, So, C.F. = C1e--z + C2e5z
a0.x.n -+a 1x
11- l
--1 1
Y -- F( x)-
- X
dx" dx"- dx 1 2
is known as Cauchy-Euler Equation or is called and P.I. = (D1 - 5) (D1 + 1) e z sin z
Homogeneous linear differential equation. Where a0, al'
a 2 , .. . , a,, _p a,, are constants and Xis a function of x only. 2z 1 .
=e sm z
Working-Rule for Solving the linear Homo- (Di + 2 - 5)(DI + 2 + 1)
geneous Differential Equation
Let = e2z - - - -1- - s m.
z
(D1 - 3) (D1 + 3)
a0 x" lY' + a 1 x"""' 1 Dn-1 + ... + a,,_1 x D + a,, =X
be the homogeneous linear differential equation. = e2z 1 Sill
. Z
Step I: Put x = i' or z = log x, x > O D12 -9
~ d d e2z 1
Mep II: Assume D = - and D = - P.I. = - - sin z =-- x 2 sin (log x)
1
dz dx 10 _ 10
xD=D I The required solution is, y = C.F. + P.1.
x/Y = D 1(D1 - 1) l

x3D3 = D 1(D1 - 1) (D1 -2) Y -- C 1X-I + C r 5 - IO1 X


2 . (I
sm og x)
where C1 and C2 are arbitrary constants.
·························································· 1
x"IY'=D1(D 1 - l)(D1 -2) ... (D -n) Example-53: Find P.L of x- 1
1
Step III: The general solution is, (xD+ 1)
y = $(z) Solution: We are given that the equation,
Since z = log x
1 -I
The required solution is, y =$(log x) X
(xD+ 1)
Example-52: Solve: (x2D2 - 3xD - 5) y =x2 sin (log
x)
So' - - -1- x-1
p.I.-
Solution: We are given that the homogeneous linear (xD + 1)
differential equation, Put xD = DI and x = e=
(x2D2 -3xD - 5)y = x2 sin (log x)]
1
Put x =i' or z = log x P.1. = - e-z
and x2D2 = D1(D1 - 1), xD = D D1 +1
(D, (Di - l) -3D 1 -5)y= e2zsin z 1
P.I. = ze--z = log x. _!,_ = log x
X X
:FIE~~EN:'._'.T_:IA'.::.L_:E~Q_::UA~T~IO~N~--------------------------:------.;;.6_8_7
D•F~
Equation reducible to homogeneous linear Solution: We are given that the equation, - 8( I + 2xf
forrn [(I + 2x)2 IY - 6(1 + 2x) D + 161.Y-
1
A linear dt•ffierent1a
· I equation
· of the form Put 1 + 2x = e= or z = 1og (I +2x)andaso,
,,d"y d" - 1 (1+2x)2IY=D1(D1-l)
ao (ax+ b) -,-, + a1 (ax+ b)"-1 ----1'.. + ( I + 2x)D = D1
dx dx" - 1
dy Auxilliary equation,
+a11 _ 1(ax+b) dx +a,,y=F(x) D .(D - l) -6D 1 + 16 =0
1 1
or 4(D 12 -4D 1 +4)=0
is known as legendre's linear equation.
or (D 1 - 2) (D 1 - 2) = 0
where a 0, al' · ···· ·0 ,t a, bare constants and Xis the funct·1on or 2 2
D1 = ,
ofx only. S C.F. = (C + C 2z) e'l::
Working Rule of Solving Legendri's linear equation °• 1
C.F. = [Ci + c log ( J + 2x)] ( I + 2x)
2
2

Let ao (a.x+bt D" +a1(a.x+b)"- 1D"- 1 + ... a~ I 8 2z

== F(x) =X P.I. = 4(D - 2)2 e


1
Stepl:Let ax+b=e= or z=Iog(ax+b)
2
Step II: (ax+ b)D = aD Z 2z
1 = 2- e
(ax+ b)2D2 = a 2D 1(D 1 -1) ~
2
(ax+ b)3 D 3 = a 3D 1(D 1 - 1)(D1 -2) P.I. = z2e2= = [log (1 + 2x)] 2 ( 1 + 2x)
The required solution is given by,
····································································· y = C.F. + P.I.
(ax+b)"D"=a"[(D 1(D 1 -1) .... ..D 1-(n-1)] = [C 1 + C 2 log (1 + 2x)]
Example-54: Solve: [(I + 2x}2 D2 - 6(1 + 2x) 1J + J6Jy (I + 2x) + [log (I + 2x)]2 (1 + 2x)2
2
== 8(1 + 2x}2 where C 1 and C2 are arbitrary constants,

An equation of the form, when one solution of corresponding homogeneous


y' + a 1(x) y7 - 1 + ... +a,,_ 1 (x)y 1 + a,,(x) y = r(x) equation is given, then "'!e apply this method.
is known as the non homogeneous linear equation of n th Let Ly= 0
order with variable coefficients.
Linear Differential equation of ll nd order with d2 d
L= - +
2
P-+Q
variable coefficient dx dx
An equation of the form is one part of C.F.
y" + py' + Qy = R Let z =f (x) is one of given part of C.F. and y = Vz
where P , Q and R the function of x or constant only is is general solution of the equation where V is function of
known as the linear differential equations ofllnd order with X.
variable coefficients. On differentiating with respect to x, we get
Working Rule for Solving the Linear Differ- y' = Vz' + Vz
ential Equation of I I nd Order with Variable again differentiating y" = Vz" + Vz' + Vz' + V'z
Coefficient or y" = Vz" + 2Vz' + V'z
• Reduction of order Putting the values ofy' and y" in the given equation,
• Change of independent variable we get
• Normal form V'z + 2Vz' + Vz" + P(Vz' + Vz) + QVz = R
• Variation of Parameter V"z + (2z' + Pz) V + (z" + Pz' + Qz) V= R
Rule I: Reduction of Order z is the solution of one of C.F. of G.S. so
'
Consider the differential equation z" + Pz' + Qz = 0
y" + p(x)y' + Q(x)y = (R) V''z + (2z' + Pz) V = R
MAlli

68 8 ~
(p + 3.z z·)
I?

or V" + ( P +; ')v '


z = :
which lin ea r di ff er en tia l
t '+

eq ua tio n with con <


l :::c....._

Stant
t V" = t~ we ge
t
Pu t V ' = coefficie nts.
ogeneous
e kn ow ledg e o f a solution o f the hom equar1011.
e m et ho d depe nds upon th
e abov
Rtmark : Since' th ch equation given below,
tio ns o fsu
There are some solu
(i) Let z = eax is th
e solution.
then, ax = 0
a2~ + a P ~ + Qe
2 + aP + Q) = 0
~ (a
eax ;t: 0
(a 2 + aP + Q) = 0,
(ii ) z =>..n, is a solu
tion, then
11 - 1 +Qx"' = 0
m(m-1)x111 - +mx'
2
+ Qx2] = 0
x111 - 2[m(m - I) + Pmx
or x2 = 0
m(m - 1) + Pmx + Q

f dV
e general solution o X- = C I
Example-55: Find th
-J )y ' + (x -1 ) y =
0 rJx
xy" -( 2x
lution. dx
ifey is one o fthe so dV = C1 -
n that the equation,
Solution: We are give
x
- I) y = 0
xy" - (2x - 1) y' + (x V = cl log X + C,
or -
Integration constant.
where C1 and C, are
or
y"-(2-})y1+(1-;)Y =O Ex ample-56: Fi nd the genera l solutio
n o the equation
f
+ P y' + Qy = R,
ven equation with y"
On comparing the gi
we get Y -
2
, ( x2 ; (,ry' - Y)
+/) =0

pa -(z-~) and Qa(l-;}R aQ


ify(x)
ven that
o fthe given equatio
= x is one o fsolution equation,
n.

Solution: We are gi

1 +P+Q= l-2+!..+1-!.X. =O
X , (2 ) , ( 2+ ;I) y = 0
y - ; +l y +
x2

given equation is z =
rr, then with
so, if one of the solutio
n of the
m paring the given differential equation
On co
V"+( +; )v' = :
P z y" + Py' + Qy = R,
we get

a{~+ J. +~J. R Q

v· +e-2+ ,: )v- ao
I Q{~
a
p
e'
is z =x, then
of the solution of the given equation
if one
V"+!..
X

Put V ' = t' and V =


v•=o
t we get
v·f+;,}'-:
, I
t + - t = 0 or
dt 1
- +- 1 = o
X rJx V " +[ -1-~+; V
? '= O 21
X
dt dx V '- V = O
or -l + -X= o t, we get
t Put V ' = t' and V =
On Integrating, we ge dt
Xt = cl or XV =C
I ( - t = 0 or - -
t =0
dx
DIFFERENTIAL EQUATION

dt 689
, dx 0 Examplc-~7: Solve (I t x 1)1 y'' , 2x(/ + x 2) Y 1- 4y ~ ()
On Integrating, we get, Solulion: We arc given that the equation,
( I ·t x 2 ) 2 y'' + 2x( I + x 2 ) y' + 4y = 0
log t -x = C
I 2.r 4
ory" + - - y'+ - -- - y . c= ()
2
l +x · ( l +x 2 /
dV On comraring the above equation with
- = e·• ·• c,
dx
7. " + p7. ' ) Q I<
Yi ➔ ( ,2 Y1 + -y Y .2
dV = ex+ c, d.x 7. z z
4
again Integrating, we get, v = e·•· -~c, + C
2 (I+ x 2 ,2
we get, - - , 2~ =4
or z

or y= x[C2 +ex+c, or z'2 = - -2- or 7.' = --.,


1 (I+ x ;2 I + x-
where C 1 and C., are the constants of. t .
f 1 -d in egrat1on or z = tan I
x
Change o n epe~dent Variable ·
If the method of reduction of order f; ·1 S . z" = -2x
change of independent variable x intao1 s. omet11~es the implies
r. • · a new variable (I+ x 2 )2
may 1ac1htate the solution of the equat· z,
" wn.
Let y + p ' + Qy - · -2x 2x I
. . y - R be the non-homogeneous - --2 + · - --=-
Jrnear equatwn. z"+ pz' (l+x )2 (l+x2) (l+x2 )
and = 1 =O
Let z =f(x) z ,1
(I+ x 2 )2
dy dy dz
dx = dz . dx or y' = y,z, On putting the above values in the given equation,
we get
, . dy d)' y 2 +4y= 0
Here y =- andyI = -
or (.D2 + 4)y = 0
d.x dz
2 The auxilliary equation is,
again, d y = .!!:.._ (dy. dz) m2 +4 = 0
dx 2 dx dz d.x m =±2i
2 So,
d y d y =C 1cos2z+C sin2z
dx2 = d.x (y . z) or y = C 1cos 2(tan-~ x) + C 2 sin
2(tan- 1x)
where C 1 and C 2 are the constants of integration.
y" = .!!:.._ ( dy ) dz + .!!:.._ ( dz ) dy
dx dz d.x d.x d.x dz Example-58: Solve: cos xy" + sin xy' - 2 cos3,r y = 2
cos5x
2 Solution: We are given that the equation,
" = .!!:.._ ( dy ) dz + dy d z
cos J..)1 11 + sin xy' - 2 cos3 xy = 2 cos5 x
2
y dz dx d.x dz dx or y" + tan xy' - 2 cos2 xy = 2cos4 x
2 2 2 On comparing the above equation with,
"=· d y (dz ) + dy d z
Y dz 2 dx dz dx 2 z" + Pz ') Q R
Yi +( ~
l.!.
Y1 + -~ Y = -
7 ~ ,1.
1, +
Z ~ z
or y N == y 2 z- Y 1Z
ll

.
On Putting the value ofy' and y" in given equat10n, - 2coi/ x
we get, --J.- - = -2
we get z
YzZ ,2 + Y i z" + P(yiz) + Qy = R or z' = COS X or z = s in x

z"+ Pz ') Yi + , y - }!_


Y2 + ( - -- a (R) _ or
z"+ z'
z" = -sin x
-sinx+cosxxtanx
za z"' z and - - = - - - -- ~- -
za (cos x)2
690 '
MATHEMA,tcs
z" + z' =
0 Example-59: Solve: y" + 2xy' + (x 2 - 8)y = x 2 e-x212
z,2
On putting the above values in the given equation, Solution: We are given that the equation,
we get
y" + 2xy' + (x2 - 8)y = x 2e-x'l2
y? - 2y = 2 cos 2 x
or (D2 -__2) y = 2 cos 2 x = 2(1 - sin 2 x) On comparing the above equation with the equatio
(D2-2 )y = 2(1 -Z2) y"+P y'+Q y=R n
Auxilliary equation, We get
m 2 -2 =0
P= 2x
-J 2x
m = ±✓2i and z = e "Tdx
So, C.F. = C1 e✓2z + C2 e-✓2z or z = e-x212
and P.l. l 2 and l 2 l dP
= 2 -2- (l-z) I= Q--P ---
(D - 2) 4 2 dx
P.l. =z2
P.l. = sin2 x = (x 2 -8)--l(4x 2)--l x 2
Hence , the required solution is, 4 2
y= C.F. + P.l. =x2- 8-x2 -1

Y = Ce✓
I
2z +e e-✓2z +sin 2 x
2 or I=-9 , S= -=x R 2
or y = C1 cos h✓2(sin x) + C sin h✓2(sin x) + sin 2 x z
2 Then, V'-9V =x2
where C1 and C2 are the constants of integration.
or (D 2 -9)V= x 2
Norm al Form or (Cha nge of Depe nden t
Variable) Auxilliary equation,
The equation transforms to the normal form by m2 - 9 = 0
the
removable of first derivative which may be achiev m=± 3
ed by So,
the suitable choice of the dependent variable. C.F.= Cle3x + c2 e-3x
Let the new dependent variable be v and u be some 1
function of x, normal form is, and P.I.= - - x2
(D 2 -9)
V"+I V=S
where ,

I= Q - l_ p 2 _ l_ dP = -t[1- ~2r x2
4 2 dx
2
R U= = -¼[1 + ~ ..... .]x2
S= - e-J f 11x =z
u
The non-homogeneous equation is, 2
y"+P y'+Q y= R = -¼[x +¾]
Let y = vz Hence
On differ entiat ing/= V'z + Vz' V= C.F. + P.I
Again Differentiating, we get
V= C1e3x + C2 e- 3x _ ?
y" = V'z + 2v'z' + Vz" 91[x- +92]
On Putting the values of y' and y" in the given
equation, we get So, y= Vz
V''z + 2V'z' + Vz" + P(y'z + Vz') + QVz = R

or y= [ Cie3x +Cze- 3x -9I ( x2 +92 )~If__/r.


V" + (P + 3!::_)v '+ .!. (z" + Pz' + Qz)V = Ii
z z z
2 ,
2z' dz I -
y = c1e3x-x 12 + Cze -3x-x-12 _ 9I(x2 + 2) e-/12
and P +- 2
= 0 - = -- P dx z = e-J!.dx 9
z z 2 where C 1 and C2 are arbitrary constants

,.
691
DIFFER ENTIAL EQUAT ION

Example-60: Solve : Thus y= Vz


2 2
log x x (log x)
2 , 2+lo gx-2---"- x/ -
(log-'--y
or y=C 1x 2 logx +C -x-+
2 3
,, - - - - y + --...c _--'-
2 (log x/ - log x
y x log x x
on, where C 1 and C 2 are arbitr ary const ants.
Solution: We are given that the differ ential equati Varia tion of Para mete r
2 , 2+lo gx-2 (logx )2 limen tary
,, --'"--' ----y = log x This metho d can be applie d only when the comp
y - - - - Y + -----'- :------
x log x x 2 (log x)2 function of the given equat ion is know n.
y"+P y'+Q y=R ... (i)
on,
On comp aring the above equati on with the equati · the soluti ons of the corre spond ing
y" + Py' + Qy = R, we get Let U and Vare
- f-2-d x homo geneo us equat ion
z= e 2xlog Lct y=A U+B V
P= _2_
xlogx ' On Differ entiat ing,
f Id, we gety' =A'U + AU+ B'V+ BV
z= e , y" =A" U+ 2A'U +AU' + B'V + 2BV + BV'
z = elogt = t Now choos e A and B as
A'U+ B'V= O
z = logx y'=A U+B V ... (ii)
Thus
I 2 I dP and y"=A U'+ A'U+ BV'+ B'V
I= Q-- P - - - ... (iii)
4 2 dx
value s of y' and y" in the given
4 On puttin g the
2+ log x-2(l og x)2 _ _!_ _ _ _ equati on, we get
2
x (log x)2 4 (x 2 log x)2 A'U' + A'U + BV' + P(AU + BV)
+Q(A U+B V)=R
l 2(
+-· l 2 ] A'U + B'V + A(U' +PU + QU)
2 (x log x) + B(V' + PV + QV) = R
A'U + B'V = R
-2
I= , VR VR
x- A'= -
UV'- U'V w
R logx
S= - = - - = l UR UR
z logx B'= =-
UV'- U'V W
,, 2
N ow, V - 2 V =I u V w
X
2 and W= lu · V' W'
x 2 V' - 2V= x
U" v .. W'
Putx2 V' = D 1(D- 1) andx = e"or U= logx,
we get and A = -I VR + C
W I
(D 1(D 1 - 1)- 2)V= ew
or (D/- D, - 2)V= ew or D 1 = 2 1 - I UR
B=
So, C.F. = c,ew + C2e-U I -w+ C>-
2U 2
1 Exam ple-61 : Solve : x y" + .,y' - y = .,~e,·
_l _ _ ew e
and P.l. 3(D- 2) Solut ion: We are given that the equat ion,
(D- 2)(D + 1) x 2y" + xy' - y = x 2 e-'"
Uew The corres pondi ng homo geneo us equat ion is,
P.r. = -3 - x 2y" + xy' - y = 0
which is Cauch y equat ion,
U w
V = C 1e2u + C 2 e -U +-e lj
Hence 3 Putz = logx and D = -d
1
2 dz ,,
v= C -~ x It reduc es to,
or ,x2 + -C2+ -x-log
X 3 (D 1(D 1 -I)+ D 1 - l)y =O
I

I
I
I.
,ii
692
MATHEMAT
· . ---ICs
(DI -1) (DI+ 1)y= 0 Sbu st1tutmg the values of y' and ,, 10
. th ---
Auxilliary equation is, equation, we get Y e given
(m-I)(m+l)=0
2 , V,
m= 1, -1 x 0 - + -2V + xU - -V - xU - -V
X X X =x2eX
So, C.F. =cl~+ e2e-z
or x 2 U- -V =x2eX
or C2
C.F. -_ C1 x+- ---Oi)
On solving equation (i) and (ii) we get,
x
Now, replacing the parameters C and C by the
1 2 U = _!_ ex V '= - _!_ x 2 ex
variable U and V respectively, the general solution of the
2 ' 2
given equation,
ex
V U= -+C1
y= Ux+- 2
x
On differentiating w.r.t. to 'x'
and
we get V' V v={-x: +x-l}'+c,
y'= U'x+-+U--
x x2 The required solution is given by,
V' y=
Choose U'x+- =0
X 2
ex ) ( x ) ex C2
, V ( --; + C1 X + - + X-1 --; +--;
then, Y =u-- 2
... (i)
x2
Again differentiating w.r. to 'x' we get C X
or y= C1x+2+ex -~
X X
V' 2V
y"= u'-+- where C 1 and C2 are arbitrary constants.
x2 x3

CHECK POINT EXERqSE •_:_ 14,. 3.


I.Find the general solution of the following equations:
(i) 8y" - 2y' - y = 0 5. Show that if auxilliary equation of
(ii) y" - 8y' + 16y = 0 y" + ay' + by = 0 has no double root then y = xeh is
111
(iii)/" - y -
9y" - I Iy' - 4y = 0 not its solution.
(iv)/" - a4y = 0 6. Show that the derivative of any solution of
2. Solve the following initial value problems:
y" + ay' + by = 0 is also a solution.
(i) y" - 4y' + 3y = 0, y(0) = -I , y'(0) = - 5
7. Solve the following ,
(ii) y" - 4y' + 4y = 0, y(0) = 3, y'(0) = I
3. Solve the following boundary value problems: ·1 (i) y" - 2y' + 2y = 2eX COS X
(ii) y" + 4y = 4 tan 2x
(i)y"+y=0,y(0)=2 , y(f )=-~ 8. Solve:

(ii) y" + y' - 2y = 0, y(0)


(i) x2y" + xy' - 4y = 0
= 0, y(I) = e - e- 2
4. If a > 0, b > 0 and y(x) is the general solution of (ii) x 2y" + Sxy' + 4y = 0

y" + ay' +by = 0. Then find lim y(x).


X ➔ = (iii) x3y'" + 2x2y" + 2y = 10( x + ~)
SOLVED EXAMPLES
Example-I. Solution of (I + y 2) dx = (tan- 1y-x)dy is
(a) x = tan- 1 y - l + ce-ian - i Y (c)x= tan-I y+ce-1an•1Y
(b) y = tan- 1 x - 1+ ce-ian-i x
(d) y = tan - I X + Cetan-l .r
DIFF ERE NTIA L EQU ATIO
N

Sol utio n: (a) Giv en, differen


tial equation is
(1 + y2) dx = (tan- 1 y - x)
dy Solution: (a) Let x2 + y2 = t
693
whi ch con tain s y2 and tan- •y
y but sinc e onl y x occ urs, we and is therefore not linear in Differentiating w.r.t. x, we get
hav e
2x+ 2yd y = dt
2 dx
(1 + Y )- = tan- 1 y-x d.x d.x
dy . y dy
implies I dt
-- = --- 1
dx X x d.x 2x d.x
⇒ -+- - =tan- ly
dy
-- Therefore, the given equation
bec ome s
l+ Yi l + y2
wh ich is line ..]_ ~ - I + 2t - I =0
Inte grat ing fact or is ar diff ere ntia l equ atio n. Hen ce , 2x d.x t +I
implies I dt 21- I
1- - -
IF = ef P(y)d y = e f -I+'-y2 dy = etan- 1Y 2x d.x / +I
Hen ce, the solu tion is
2-/
ytan- I t+I
x · (l.F .) (I.F. ) dy + C
2
= J- -
I+ y implies 2xd x = ~dt
1
2-1
imp lies - I
xe tan- y = J- Y ,an- Ydy + C
tan--e 1

I+ y 2 implies 2xd.x + (1 + - 3-)d r = 0


Let t-2
tan- 1y =t on integration, we get
The n x 2 + t 3 lof
____!:z_=dt implies 2x2 + y2 + 3 log (x- (t - 2) = C
1+ y2 which is the required solution + _1,,2 - 2) = C
.
So n etan ~ly Hence option (a) is corr ect
Exa mpl e-3 .. The par ticu lar
y =~C OS Xis
integral of ([Y - 2D - 4)
f te' dt + C (a) cos x (b) sin x
= te' - J1 · e' dt + C I x
(c) (d) 1 ex sin
2 e cos x :c
= te 1 -e1 + C Solution: (c) Part icul ar inte 2
gral
= (tan -I y -1) etan-i Y + C
imp lies X = tan-l y- 1 + ce_,an- i y = ( D~ - ~D _-+ }e' .: os :c )

whi ch is the req uire d solu tion = e 1


. ,
Hen ce opt ion (a) is cor rect (D + 1)- -2( D + l)+- + cos x
Ex am ple -2. The sol utio n
of diff ere ntia l equ atio n
y(x 2 + y 2 + J) dy + [2x (x 2 + t' ' - ,-
l - i.'.'\."S _\
y2) - J]d x = 0 is o- +_,
(a) 2x 2 + y
+ 3 log (x 2 + y2- 2) = C
(b) 2x2 - , l
y-
3 log (x 2 + y 2 - 2) = C = t' - - - 1,.' VS \
(c) 2x2 + - l + _;
y
+ 3 log (x 2 + y 2 + 2) = C
(d) 2x2 - y- l ,
= ~ c' l'l)S . \
3 log (x 2 + y 2 + 2) = C
Hen ce option ld is ~·l1rr~1:t

EX ER CIS E LE VE L-I
· 2.
1
Let k be real constant. The solu One of the integ ratin g t,Ktl,rs
tion of the differential ,,tth e diile rent ial equati,:m
.
equatio dy dz 3
l1-' .h y)dy t lx 2 xy), /\· =
u is
ns - =2y + z and - = y satls . ti th .
1es ere Iat1on l
dx dx (u) 2 !
(a) y - z = ke 3x X .\
lf,) -+
(b) y + === ke x ,-, .
(c) 3y + z == ke 3x 3 I
(d) 3y + z == ke- x (c) lxv) 2 l<IJ
.\ _\'
__ __ __ _ ~--:-----:-~--=-:-:=-::::=~====M:-A:--T-:-H~~
arly ind epe nde nt solution
of
4!.___ __ __ __ __ __ __ __ __ 11. Let y (x) and yi(x ) be line
6~9~ 2yd x- (3y - 2x) dy = 0 ) y' + Q(x )y == 0, where
atio n n y" + P(x
•'· The diff emt ial equ the diff eren tial equ atio rval/.
but not line ar ous fun ctio ns on an inte
(a) exa ct and hom oge neo us P(x ) and Q(x ) are con tinu
line ar but not exa ct + by 2 (x) and yix ) == ey 1(x) dy 2(x) are
(b) hom oge neo us and Th eny lx) == ay 1(x) tial
line ar but not hom oge neo us tion s of the giv en differen
(c) exa ct and line arly ind epe nde nt solu
and line ar equ atio ns if
(d) exact, hom oge neo us n
n of the dif fer ent ial equ atio (a) ad= be (b) ae == bd
4. The sol utio
2 2 0 . (d) ac::;: bd
+ y -1 ) (O) -_ 1 IS (e) ad::;: be o,
dy -_ -~
-
x(x--- =- --- ,y ntia l equ atio ns , sati sfyi ng y(O),,,
2 2 +5) Con side r the diff ere
dx y (x + y 12.
4 2 4 20x2 - !Oy2 + 11 = 0
O, ...1a I va Iue problem
(a) x - 2x y2 - y - x:o;O . T 11·1s m1t
4 2 2 + y 4 + 20x 2 + !Oy2 - ll = 0 whe re <j>(x) = { I, x> O
(b) x + 2x y
4 2 4 2 - !Oy2 + 11 = 0 tiable
(c) x + 2x y2 - y + 20x tion wh ich is not differen
(a) has a con tinu ous solu
4 2 4 - 20x 2 + I Oy2 - 11 = 0
(d) x + 2x y2 + y atx =O
dx - xy sin le at
tial equ atio n 2 cos (y) tion wh ich is differentiab
S. Con side r the differen (b) has a con tinu ous solu
(y) dy = 0
X == 0
fact or le on R
(a) e-'· is an inte gra ting tion whi ch is differentiab
an inte gra ting fact or (c) has a con tinu ous solu
(b) e-x is tinu ous solu tion on R.
fact or (d) doe s not hav e a con
(c) 3x is an inte gra ting n
3 fact or of the diff ere ntia l equ atio
(d) x is an inte gra ting 13. The ~ar ticu lar inte gra l
2 / The n
I, 1] ➔ R, f (x) = x 3, g(x ) = x / x + 3) is
6. Let_/, g :[- y " + y' + 3y == 5 cos (2x
rly ind epe nde nt on [- 1, I J
(a) f and g are I inea (2x + 3)
(a) 2 cos (2x + 3) - sin
dep end ent on [- 1, 1]
(b) /an d g are linearly (2x + 3)
on [- 1, l] (b) 2 sin (2x + 3) + cos
is NO T Identically zero
(c)f (x)g '(x) - /(x) g(x ) q(x ) suc h 3) - 2 cos (2x + 3)
ous fun ctio n p(x ) ai1d (c) sin (2x +
(d) There exis t con tinu + 3)
tha t/an d g sati sfy 3) - cos (2x
(d) 2 sin (2x +
1, I] of diff ere ntia l equ atio n
y" + py' +q y = Oo n[- The gen eral solu tion
equ atio n 14.
The sol utio n of the dif fer ent ial 2 0 is
7. 4x y" - 8xy ' + 9y ==
y y) y y = O wit h init ial
( x co s-d X (a) cle 5x!2 + c2e -3xl2
x sinX- -y cos -X dx+
con diti on y (O) = 0 is (b) ele 3x/2 + e2e -3x/2
312
(e) (e 1 + c 2 log x) x
(a) x/ sin.; I= I (b) y = ± nrr.x
(d) (clx 312 + cze -312)
2 ord er linear
solu tion s of a sec ond
15. Let 1, x and x be the < x < I.
(c) y = x/ sin ; I (d) x = y eren tial equ atio n on -1
non -ho mo gen eou s diff tc111ts
, inv olv ing arb itra ry cons
The n its gen era l solu tion
=xe-2x is as
x dy + (3x + I) y 2
c and c , can be wri tten
1
8. An inte gra tion fact or of dx
2) + I
3 (b) 3xe-'· (a) ci(l -x) + ci( x-x
(a) xe x 2
(d) x 3e-r (b) c 1x + er + 1
(c) xe-'
2)+ J
2 . .(c )e1 (I +x )+ ci( l +x
. f x 2 -d y - 5x -dy + 9 y == 0 1s 2
9. The gen eral solu tion o 2 (d) cl + e2x + x
dx dx
3 erential
(a) (e 1 + er )e-'' (b) (e 1 + e 2 lnx )x )dy == O is an exa ct diff
16. If g(x , y) dx + (x + y tion of
2
2 the gen era l solu
1
== x , the n
(d) (c 1 + c 2 In x) ex equ atio n and if g(x , 0)
(e) (e 1 + e 2x)x equ atio n is
the diff eren tial
(He re e 1 and c2 are arb
itrary con stan ts .) c
eren tial 3 (b) 2x 3 + 6xy + 3y2 =
eral solu tion of the diff (a) 2x + 2xy + y2 == e
IO. lf(e 1 + c1 In x)!x is the gen + y2 == c
dy I 2
(d} x + 2xy
• ? d 2y (c) 2x + 2xy + y2 == c
> 0 then k equ als equ atio n
~ + k.x - + y == 0, x 17 sol uti on of the dif fer ent ial
equ atio n x- -de dx • A gen era l
(b) -3 d 3y d2y
(a) 3 + 4 y == 0 is
(d) -I - - 3 - -dx2
(c )2 dx3
DIFFERE NTIAL EQUATI ON
695
lO) y = c l e' + c2e2x + cre2x Genera l solutio n of the differe ntial equati on
24.
(b) y = c 1e- x + c 2e2x + c xe 2x xdy = (y + xe-ylx)d x is given by
3
(c) y = c e- x + cre- x + c e 2 ' (a) e ylx= In x + c (b) eyl.< = In x + c
1 3
(d) y = c2e- x + C2e' + C3e4x ( c) e- xly = X + c ( d) e''Y = X + C
The Wrons kian of the functio n/1(x) = x2 and/i( x) = x Ix
25.
18. Consid er the differe ntial equatio ns dy _ y = -y2. I is zero for
dx
(a) all x (b) x > 0
Then lim y(x) is equal to
X---+ DO (c) x < O (d) x = 0
2
(a) - 1 (b) 0 26. The genera l solutio n of y'' - m y = 0 is
(c) I (d) (a) c sinh mx + c cosh mx
oo 1 2
(b) c cos mx + c 2 sin mx
J9. Consid er the differe ntial equati on : 1
= ay _ by2 , where (c) c cos mx + c 2 sinh mx
1
a, b > 0 and y(O) = Yo· As x ➔ + 00 , the solutio n y (x) (d) c 1 sin mx + c 2 cosh mx
tends to 3
27. The orthog onal traject ories of the curves y = 3x + x + c
(a) 0 (b) al b are
(c) bla (d) Yo (a) 2 tan- 1 3x + 3 In I y I = k
20. Orthog onal traject ories of the family of curves (x -1)2 + (b) 3 tan- 1 3x + 2 In I y I= k
y2 + 2ax = 0 are the solutio n of the differe ntial equatio n (c) 3 tan- 1 3x - 2 In I y I= k
(d) 3 In Ix I - 2 tan- 1 3y = k
(a) x 2 -y 2 -1+2 xydy = O(b) 28. The genera l soluti on of ·the differ ential equat
ion
dx 2
(6x - e-y 2 )dx+2 xye-y2 dy=0 IS •

X
2
+ )' 2 - 1 + 2xy -dy = 0 (a) x2(2x - e-y') = c
dx
(b) x2(2x +e-y2 )=c
(c) x - y - I - 2xy -dx = O(d)
2 2
2
dy (c) x (2x + e- y2 ) =c
(d) x(2x2 -e-Y )=c
2

x - + y 2 + 1 - 2xy -dx = 0
?

dy 29. The solutio n of the initial value proble m xy' - y = O with


y( l ) = 1 is
21. The solutio n of the differe ntial equati on
=x 1
d 3y dy . (a) y(x) (b) y(x) = -
- - - 9 - = COSX JS X
dx 3 dx
(c) y(x) = 2x - 1(d) y(x) = _ l_
(a) y(x) = C e 3x + C e 3-' + C +_!_s in x 2x-l
I 2 3 }0
30. If Y[ (x) = 3~ 1 (x) + 4y2 (x) and y~ (x) = 4y (x) + 3y (x)
(b) y (x) = C e 3x + C2 e-3 + C3 theny l (x) JS
1
x _ _!_sin x I 2 •
10 (a) c le-~ + c2e7x (b) c e-~ + c e- 7x
I 2
(c ) y(x) = C e 3x + C,e- 3x + C +_!_c os x (c) c l e - x + c2e7x (d) c l e- x + c e - 1x
1 - 3
10 31. If e-' + ~Y + x s_in Y + eY = c is the ~enera l soluti on of an
1 exact
is d1ffere nhal equati on then the d"ff, .
(d)y(x ) = Ce 3 x +c e- 3 x +c - - cosx ' .
1 erenti al equat ion
I 2 3 10
22. Consid er the differe ntial equati on y" + 6y' + 25y = 0
with initial condit iony(0 ) = O. Then, the genera l solutio n (a) dy = ex - y - sin y
dx ey - x - x cos y
of the IVP is
(a) e- 3x (A cos 4x + B sin 4x) (b) dy = e-' + y+sin y
(b) Be- 3x sin 4x dX y
e + X + XCOS y
(c) Ae-4x sin 3x
(d) e--4x (A cos 3x + B sin 3x) (c) dy = - (e"' + Y + sin y)
23. The solutio n of the differe ntial equati on y" + 4y = 0
dx e Y + x + x cos y
subjec t to y(O) = I , y' (0) = 2 is
d
(a) sin 2x + I (b) cos 2x + 2x (d) -1.... = -(e X
- Y - sin y
(c) sin 2x + cos 2x (d) sin 2x - cos 2x dx ey - x - x cos ~
696
~;---::-~-----------
32 --------------------- ---...:M.:.:::_~"11 es
· The solution of the differential equation (h) The differential equation is exact
2 (c) e-' ' Y is an integrating factor of the differe .
(x y + xy2) dx +( ,~' + .,:2y + sin y ) dy = 0 is equation nt1a1
(d) A sui~able substitut_ion transforms the differenti
x3y x2y2
(a) - + - · - - c o s v=c equation to the variable separable form able
3 2 .. 40. Which one of the following differential e .
x3 :i.:2 ,2
represents all circles with radius a? quations
(b) ~ + _· _)_ + cos y = C 2 2
3 2 ·
(a) I+ ( d ) Jx. + ✓a 2 -x 2 ddx; =O
X3 X2 )' 2
(c) - + - - - c os y =c 2 2
3 6 d )
(b)l+ _)'_ + ✓a 2 -y2___l_
d =O
3 2 2 ( dx 2
X X )' dx
(d) - + - - + c os y=c
3 6 ·
33. The differential equation (2x2 -~ by2)dx + cxydy = 0 is (,) HtH+a'(~; J =0
made exact by multiplying the integrating factor __!_ .
x2 2
Then the relation between b and c is (d) [1+(::Jr =a ( : : ; J
(a) 2c = b (b) b = c
41. Solving by variation of parameter y" - 2y' + y = e'logx,
(c) 2b + c = 0 (d) b + 2c = 0
the value ofwronskion Wis
34. If general solution of the differential equation ay"' +
(a) e211 (b) 2
by"+ cy' + dy = 0 is linearly spanned by eX, sin x and cos
x, then which one of the following holds? (c) e- 2,- (d) None of these
(a)a+b-c-d=0 (b)a+b+c+d=0 42. The p-discriminant of the equation
(c)a -b+c-d=0 (d)a-b-c-d=0 1 .
y= px+- IS
35. Ify = In (sin (x +a))+ b, where a and bare constants, is p
the primitive, then the corresponding lowest order
differential equation is (a)y2=x (b)y=x
(a) y" = - (1 + (y')2) (b) y" = y 2 - (y') 2 (c) y2 = 4x (d) None of these
(c) y" = 1 + (y') 2
43. The c-discriminant of the equations
(d) y" = y' + y 2
36. (y- c)2 = x(x - a) 2 is
The solution of the initial value problem
(a)x(x-a 2 )=0 (b)x-a2=0
dy sin x .
- = - - , y(0) =0IS (c) x = a (d) None ofthe-se
dx y+2 44. ifyi(x) andyz(x) are solutions ofy" +x2y' -r ll -x)y= 0
(a) y(y + 2) = 4(1 - cos x) such that
(b) y2 = 2(l - cos x) y 1(0) = 0, y' 1 (0) = I
(c) y(y + 4) + COS X = 0 y',(O)= l , .h (0)= l
(d) y(y + 4) = 2(1 - cos x) then the Wronskian -W(), l' y 2) on R is
(a)0 (b)l
37. . . fy' (c) -1 (d) 2
The differential equation -d2y + y = O sat1s mgy
dx 2 45. If y 1 and y 2 are tv,o solutions of initial value probkmr•
(0) = 1, y(n ) = 0 has + p(x)y' + q(x)y = 0. y(x 0 ) = y • y'(x0 ) = r 0 and the
0
(a) a unique solution Wronskion W(J,l' y 2 ) = 0. then y 1 and_\·~ are
(b) a singly infinite family of solutions (a) linearly dependent
(c) no solution (b) linearly independent
(d) a doubly infinite family of solutions (c) proportional (d) None of these
38. Two linearly independent solutions of the differential 46. Which of the following transformation reduce th e
equation y" - 2y' + y = 0 are y 1 = ~ and y 2 = xeX. Then a
particular solution of y" - 2y' + y = ~ sin x is differential equation d: +.: . log : = :, (log : )~ inll1 the
(a) y 1 cos x + y 2 (sin x - x cos x) dr .r .c
(b) y 1 sin x + y 2 (x cos x - sin x)
. du
(c) y 1(x cos x - sin x) - y 2 cos x form - + P(x) (11 ) = Q(x)
(d) yi(x sin x - cos x) + y 2 cos x d.r
39. Consider the differential equation (x + y + I) dx + (2x + 1
2y + I) dy = 0. Which of the following statements is (a) 11 = log:: (b) 11 = --
log ~-
true?
(a) The differential equation is linear (c) 11 = e= ld) 11 = (log ::) 2
697
D1FFERENTIAL EQUATION

. d4 y 54. The singu lar so lution of the differential equation


4 1. The genera I so I ut,on - d + y =0 is p 3 + px - y= 0 is
4 3
'.,\"
(a) 4x3 + 27y2 = 0 (b) 27x + 4y2 = 0
2 3
2
(c) 4x + 27y = 0 (d) 4x + 27y3 = 0
2 3
55. The value of Wronskion W(x. x , x ) is
(a)y= exp( )2 )[c, cos Fz +C 2 Ji] 2
(a) 2x 4 (b) 2~
(c) 2x 3 (d) None of these
56. Solving by variation of parameter for the equation y" + Y
-XJ[
+ exp ( ✓2 C3 cos ✓ 2
X2 + C4 sin ✓X] = sec x , the value of Wronski on is
(a) I (b) 2
( b) y= c l sin X + c2 cos X + C3 sinh X + C4 cosh X (c) 3 (d) 4
(c) y = C 1e-' + C2e-x + C3e- 2,· + C4e 2'· 57. The eigen values for the boundary value problem x" +
(d) y = c, sin 2.x + c2 cos 2x + C3 sinh 2.x + C4 cosh x Ax= 0; x(0) = 0, x(n) + x' (n) = 0 satisfy
(a) 11. + tan 11.n = 0 (b) ..ff.. - tan A7t = 0
48. dy = J + y2, y(0) = 0 in the domain R : Ix I < 5, I y I < 3
dx (c) ..ff..+ tan fin= O (d) 11. + tan JX°n = 0
the equation has 58. Suppose y/x) = x cos 2(x) is a particular solution of y" +
(a) no solution ay = - 4 sm (2.x)
(b) unique solution for Ix I < 0.3 Then, the constant a equals
(a)-4 (b) - 2
(c) has infinite number of solution
(c)2 (d)4
(d) nothing about solution can be concluded 59. All real solutions of the differential equation y" - 2ay' -
49. For TVP by= cosx (where a and bare real constants ) are periodic
y ' = 2y 112 , y(0) = 0 if
Which one is correct in a nbd of0? (a) a= 1 and b = 0 (b) a= 0 and b = I
(c) a= I and b 0 (d) a= 0 and b a: I
*
(a) It has unique solution 7
60. Integrating factor of (x y 2 + 3y)d-,; - ( 3.--,:Sy - x )¢i· = 0 is
( b) It has no solution x"'y'', then
(c) Solution exist but not uniquely (a)m=:-7 ,n=2 (b)m=-l , n= 7
(d) None of the above (c) m = -7, n = I (d) m = -7. n = -1
2
50. Let y 1(x) and yi(x) be two solutions of (I - x) 61. Let a, b E R. Let y = (y 1,y,)T be a solution of the svste m
of equations - -
2
d ; - 2x dy + (sec x)y = 0 with Wronskian W(x). If y' , =y'2 ,y2 = ay 1 ~ by,
c1x- dx
Every solution y(x) ➔ Oas x ➔ = -if
(a) a < 0, b < 0 (b) a < 0. b > 0
Y 1 ( 0) = 1 = 0 and w(½ )={ then (c) a > 0, b > 0 (d) a ' 0. b < 0
,(:to 62. The solution of dd_"
2
,. = _,- .-_,·(0) = 1. exists fo r all
dY2 J equals
x= O (a) XE (-oo, 1) E
( dx
(/,) x [0. a] where a::-- 1
(c) XE (-=. oo ) (d)x E [1. ,i]where a ' 1
(a) 1/4 (b)I
(c) 3/4 (d) 4/3 1 . . . •
d 2v so v mg
by var1at1on ot
63. For --~ +4 _,· = tan2.r
51• If e-x, xe-x are solutions of y" + ay' + by = 0, then dr
(a) a= 0, b = l (b) a= 1, b = 2 parameters. The va lue of \Vrnnskion 11 · is
(c)a=2 , b= l (d)a=-J,b = 0 (a) 1 (h) 1
52· The degree of differential equation satisfying the relation (c ) 3 (,/) 4
2
64. The initial va lu e problem
Ji+x 2 + ~ =11.(x~ - y J1+ x ) is 2

(a) I (b) 2
d v + ~+
.r 2 riv .rv = 0: _1·(0) = I, -=- (,fr)
= O has
dx d.1 d.r , ="
(c) 3 (d) None of these
(a) a unique solution
53 · The singu lar solution of the differential equation (xp - (b) no sol uti on
Y) = p 2 - I is (c) infinite ly many solutions
2
(a)x 2 +y2 = I (b)y2-x = I (</) two linearly independen t sohuion
2
(c) x 2 + 2y2 = 1 (d) x - y2 = I
698
.
6 5. MAT1-te1.1AT1cs
Consider the following statements 69. For non-homo geneous equation y' + p(x)y ~
I. A singular solution of differential equation satisfies andy2 are its solutions, then the solution ofhom ), If Y
the differentia l equation but is not a particular 1
equation y' + p(x)y = 0 is ogeneous
solution of the equation.
II. If T(x, y) = 0 is the equation of the tac-locus, then (a) Y = Y1 - Yz (b)y= l!__
T(x, y) is a factor of the P-discrimi nant. Y2
III . T(x, y) is a factor of a c-discriminant.
IV. Cusp-locu s has two distinct tangent. (c) y = fl. (d) None of these
Yi
Choose the correct answer.
. dy (1- x)
(a) Only I is true (b) I, II and IV are true 70. The solut10n of - =- - represents
(c) All are true dx y
(d) No one is true
(a) a family of circle centre at ( 1, 0)
66. The general solution dy + y = f (x), (b) a family of circle centre at (0, 0)
dx (c) a family of circle centre at (-1, 0)
x, 0 :S: x:S:l -(d) a family of straight line with slope -1
where/(x ) ={ andy(0) = 0 is
0, x ::C::l
71. Solve dy =Jiy1, for 0 < y < 10 andy(0) = 0
dt
2(1- ex), 0 :s; X :s; 1
(a) y = (a) has unique solution
{ 2e-x (e-1), x~l
(b) has no solution
(c) has two independe nt solution
2(e-x -1), 0:S:x:S:1
(b) y = . (d) has infinite solution
2e-·' (e-1), x::C::l 72. The orthogona l trajectory of the family x2 -
l y = C, are
_ given by -
2(1-e-x), 0:S:x:S:l (a) x 2 + y2 = C2
(c) y = . (b) xy = C2
{ 2e-x(l-e) , x::C::l (c) y = C2 (d) x = 0
(d) None of the above 73. A. Singular solution contains no arbitrary constants.
67. Which of the following pair of function are linear B. Singular solution can be obtained from complete
independe nt primitive.
(a) A is true, Bis false
2
x x>0 {O, x ( b) Both A and B are true
I. Y1 = - Yz = ? : C: 0
{ 0, ' X < 0' x-, X < 0 (c) Both A and Bare false
II. y 1 =x2, y2 =x Ix I, Ix I :s; 1 (d) None of the above
III. Y1= x3, Y2 = x2 Ix I, Ix I :s; 1
74. The solution of the differential equation y dy +(I+ /l
_ {<x -1)2, 0:S:x:S:l dx
IV. y 1 - , Y?
sin x = 0, y(O) = 0 exists in the open interval (-a, a) then
0, 1< x< 2' - 'a' equals to

0, o:s; x:S: 1 (a)2cos- 1 (½~) (b)2tan- 1 (½~)


={ (x-1)2, 1:S:x:S:2
(c)2sin- 1 (½~) (d)2coc 1 (½~)
Select the correct code
(a) Only I is true (b) Only II is true 75. . d endent
If y 1(x) = x and yz(x) = xe< are two linearly in ep
(c) I and llI are true (d) All are true solutions of
68. Singular solution of differentia l equation contains
2
I. arbitrary constant 2d y dy
x - - x(x +2)-+( x +2) y =0,
II. can be obtained from general dx 2 dx .
III. do not contain arbitrary constant then the interval on which they form a
IV. cannot be obtained from general solution
fundamental set of solution is
(a) I, II are true (b) III, IV are true
(a) x > 0 or x < 0 (b) - l < x < 00
(c) I, IV are true (d) II, IV are true
(c) - I < x < 2 (d) -oo < x < 00
DIFFERENTIAL EQ UATION

76. Usi~g the method of v .


particular so)uf anation of
ion to the diffi parameters C
erential equ t' or the 699
y" + 4 - 3 a ion 81. The
2 interval for x, over whi ch the ~olution of ! VP dy(2 +
Y---:----,o . n x t-y- ) - dx (I 1 2x t- :Jy ) O. y(O J O, U l x l:::: 2. iy 1
sm2r < x < -
3 . .. 2 5 1 exists, is
(a) - 2 $ X $ 2
(a) -4 sm 2x log cos 2 . - 3 (h) - I Sx -S I
X -cos
4 ' 2X·
(c) _.!_ 5x$.!_ (cf) _.!_ Sx:: ; .!_
3 4 4 2 2
(b) - sin 2x log cos 2 _ 3
2 X -cos 2x 82. Let 11 be a non-negative integer. The eigen values of the
4
3 . ·11
St unn- L1ov1 d "r
(c) . e probl em --:, + A.r = O with boundal')
2x d.c -
2 sin 2x log sin 2x _ _l xcos
2
.. ch di'
3 . cond1t1on y(0) = y (2 7t ). -=- (OJ = - I 2rr i are
(d) 4 sm 2x log sin2x-l2 X COS 2x d_r dr
(a) n (h) 112rr2
77. The general soluti (c) nrr (d) n1
on of the system f d.
dX o ifferential equation 83. The initial value problem
--:it == MX + b, where X == [ x1(t)] 2 dy
x (t) aoctM a 2 x 2 matrix (x - x) ~ = (2x - 1) y. y(x0 ) =>r, has a unique solu1ion.
2
if (x0 , y0) equals
(a) (2, I ) (b) ( I. I l
[ ~ 01] and a 2 x 1 constant vector b == [I] . .
1s given by
1 (c) (0, 0) (d) (0. I l
84. The initial value problem
(a) eM,c+b (b)e"11c + bt
dy
(c) eAft c - b (d) eAft c - bt x__:. = y, y(O ) == 0. x ~ 0 has
d.r
78. For the ordinary differential equation (a) no solution
d 2y d (b) a unique solution
(x-1) dx 2 +(cotru) ~ +(cosec 2m)y=O (c) exactly two solutions
(d) uncountable many solutions
which of the following statement is true?
(a) 0 is regular and 1 is irregular. 85. The solution of the differential l'qu:11i0n ~ - ,
i..:. r-
( b) 0 is irregular and 1 is regular.
which vanishes when x = 0 and tend5 tc, .1 ftrutc tie .'. .b
(c) Both O and 1 are regular. X ➔ -oo is
(d) Both O and 1 are irregular, (a) y = e-x + I (b) 2y = e ' - I
79. If 2x(l -y) =Kand g(x,y) =Lare orthogonal families of (c) y= e" - 1 (d) _,. = <' ' - l
curves where Kand L constants, then g(x, y) is 86. If II I' 11 2 , 11 3 are soluc i,,n , ,, r Ih a: eq u1ti,,n
(a) x 2 + 2y-y2 (b) 2{y + 2(y + (1-x))} 3 2
d y . , c/ ,· ,, ti" ,
~ + sm- x - -
·, + <'. - T \lc' f , \\ = 1 . then :: _
(c)x 2 +2x-y2 (d)x 2 -2y+y2 dx de ,LI
80. If y and y are linearly independent solutions of the au 1 + b11 3 and 011 1 T r !12 T 11, .ire .its,, s,, lut t-' tb ,, f ~1, cn
I 2 . .!' )" I + ( ) =0 Then
homogeneousequanonL(y)= y +p 1(x!Y P2 xi)' · ' differential cquatil1n . th<.' 11 th<' , .llu<.' s ,,f . 1 .mJ .~ .m:
p (x) and pz(x) are given by (a)a =- 1, !> = l lf>).i - :: . b
1
"
(c) ,, ~ 2, h
y 'i y; - y'z Yt
= Y1Yz11 - Y1YY2, p2 (x) = ~ (d) No sur h s,iluti,111 c, isb
(a) P1 (x ) w(x) 117. The larges t ,aluc ,,f , su..:h 1h,1t tht'rt' c \ 1sts .1 ti.1n..:UL'n
· <' \ '
hp ) lor ,· t h,ll .IS I .
,I S,1 ll li L1 1\
,1' .
[ r \ - I\ 1th
(b) pi(x) =[y,yz'-
w(x)y(Yz], p z ( x ) =~ ,/\
w(x)
/,(0) U is giw1 ti)

(a) !: ll> ) re
(c) p,(x) - Y.i.E-,
= Y1 Y'2w(x) pz(x)=[~ ] w( x) .I
1t
(cl (,/) re
-l
(d)p(x)= [~ 2
I w(x) ] ' p (x)==-[~ J w(x)

___,
700
-88. Let y 1 and v, be two linearly indepen dent so luti
o ns of
~~
·- 93. Lct y 1(x ) and y 2 (x) be two linearly indepe nden;:-
· --ol
.
y" + (sin x)v = 0. 0 ~ x :s; 1 · Ut1on
· . . . d2
Let g(x) = W(v 1• y 2) (x) be the Wrons kian
of the differen tial equatio n x - y - 2x 2 ~ + ,
dx. 2 dx e Y
ofy 1 andy,.
satisfyi ng y 1(0) = 1_,yif 0) = l y' (0) land /z<OJ
Then - 1
Then , the Wronsk ian of y /x) and yz(x ) at x
' 2 _, ~
(a) g ' > Oon(O , I) W( y 1 y
2 is equal to
'
)I x = 2
(b)g' < Oon[O , I] 1
(a) 2e (b) 2e 2
4
(c) g' vanishe s at only one point of [O, I]
(c) 2e 4 (d) 2e
(d) g' vanishe s at all points of [O , I] 94. The coordin ate at a moving point P satisfyi
ng th e
89. If y 1(x) and yi(x) are solution s ofy'' + x2y' - (I - x)y
=0 . d.x dy . ,
such that y 1(0) = 0, y '(0) = - 1 and yi(0) = - 1, y equatio ns - = tan x - = - ~111 - x: 1 2'.0 of the curve
1 2
' (0) = dt d1
1, then the Wronsk ian W(y" y ) on R
2 passes through the poin t (it/2. 0 ). When 1 = 0. then
the
(a) is never zero equatio n of the curve in rectang ular coo rdin ates
is
(b) is identica lly zero
l
(c) is zero only at finite number of points (a) y =-cos 2 x (b) y = si n 2x
2
(d) is zero at countab le infinite number of points
(c) y = cos 2x + I (d) y = sin2 x - I
90. For the stunn-L iouville problem ( 1 + x 2 )y" + 2xy'
+ A.X 2y 95.
2
Lety 1 andy be solution s of Bessel' s equation 1:.►
., - r-:
= 0 with y'( l) = 0 and y '(IO) = 0 the eigen values,
A + (1 2 - n 2 )y = 0 on the interva l O < 1 < x
v. irb
satisfy y 1(1)= l,y' 1(1)=0, yi(l )=O and / ( 1) = 1.then\
2 a!ue
(a) 1c ~ 0 (b) A< 0 of W{y" y 2] (1) is

(c) A:;cO (d)A$ 0 (a) u12 (b) 1/t

(c) (log t)2 (d) 0


91. Consid er the followi ng stateme nt for IVP dy t
=x2 + y ,
dx 96. Differe ntial equatio n Iy' I + I y = 0 has
y(0) = 1, D: Ix I$ 1, Iy- 1 I$ 1 (a) GS solution
1. It has a solution which exists for all n. (b) GS solution but no arbitrar y constan t
IL The local interva l for which the solutio n (c) particu lar solution which is boundn l
exists
(d) None of the above
97. The ordinar y differen tial equacio n
umquel
. y 1s. Ix I$ -I .
3
x dy - y = 2x 2 with inicial condiri ons q 0\ = 0. hl->
III. It has no solution in the interval Ix I$ 3. dx
Then select the correct code. (a) no solution
(a) Only II is true (b) a unique solucion
(b) II and III are true
(c) two distinct solu tion
(c) I and 11 are true (d) All are true
(d) infinity of so lutions
92. Let y(x) be a non-triv ial solution of the 2nd order 98.
linear The solution of y'' + ay' t- b,· ~ 0 \\ here J unJ
r JT~
differen tial equatio n co nstants , ap proal· hes 10 Lero as , -> -' ' · then
(a) a ' 0, f-, ' 0 lf-,) " .... L\ . f,-.. L\
d2 1 d (c:) u -.. 0, h -.. 0
_) +2C 2+ Ky= O l,/\ " -.. L1. f-, '· Ll .
dx 2 dx 99. Th~ primitiw uf the ditfrren tial cquJ1io11 l2-n
\" ::\\ -·
where C < O; K > 0 and C 2 > K , then v)d'< t- (.x·\ •4 ( " ., 2y 2 3.11./\ ~ Dis
(a) I y (x) I ➔ 00 as x ➔ oo
(b) I y (x) I ➔ 0 as x ➔ oo
(.u) \" 2., , t- I + - ~ :c. C lf-,) /<' ' - ~ + ~ :c l
' y r'
(c) Jim ly(x)le xistsan disfini te
J.' ---?±cn

I
(d) None of the above (_c) I \ ,' 1- - - l,/\ I','
_1·

_
DIFFERENTIAL EQUATION

100. For the initial value problem 701


y' =f (x, y) with y(O) = o, 1 1
which of the following statements is true? (a) xe ,[x x-x
- ln - - 2 ] + e- Y - ln - [-x- x- l
2 4 3 9
(a)J(x, y) = /;;; satisfies. Lipschitz's condition and
JVP has unique solution. J
(h) x e .2 .,fx2,
- 11 -
x-x2l +e · r-x
- -2, - 11 -x+- :<
2 4 2 9
(b) f(x, y) = does not satisfy Lipschitz's condition
J;;;
and IVP has no solution .2 ., x . x - x ,-x x , x
c x e - 2 In - - - +e - -.1 1n --'---
() 21 r 'J
(c)f (x, y) = I y I satisfies Lipschitz condition and IVP 2 4r _ 3 9
has unique solution
(cl) None of these
(d) J (x, y) = I y I does not satisfy Lipschitz condition. 102. All non-trivial so luti ons ofx 1y " ~ :cy ' -'- -1.xy = 0 . ,T > 0 are

2
(a) bounded and non-peri odic
101. d + P dx
Ly = xe" Lnx(x> 0) whenL = d.x d + Q ) the two (h) unbounded and non-period ic
( 2
(c) bounded and peri odic
LI solutions of Ly= 0 are xe" and e", then PI is
(d) unbounded and periodic

Answers Level I
I. (c) 2. (b) 3. (d) 4. (d) 5. (d) 8. (a ) 9. l b ) 10. rd )
6. (h) 7. (a)
11. (c) 12. (a) 13. (d) 14. (c) 15. (d) 16. (h) I 7. (b) 18. (c ) 19. ((' ) 2 0 . I ".

21. (b) 22. (b) 23. (c) 24. (b) 25. (a) 26. (a) 27. (a) 28. (d) 29. ( a l 30. 1"· 1

31. (c) 32. (a) 33. (c) 34. (h) 35. (a) 36. (d) 37. (c) 38. (c ) 39. (0 1 -10 . I d~•

41. (c) 42. (c) 43. (a) 44. (c) 45. (b) 46. (b) 47. (a) 48. (b ) -19. (C l 5-0 . I :.z I

52. (a) 53. (d) 54. (a) 55. (c) 56. (a) 57. (c) 58. (d) 59. (Cl 60. , .· \
51. (c)
65. (b) 66. (d) 67. (d) 68. (b ) 69. iU t 70. \ _; I
61. (a) 62. (a) 63. (b) 64. (a)
76. (d) 77. (c) 78. (a) 79. (J ) 80 . ,t- 1
71. (a) 72. (b) 73. (b) 74. (c) 75. (a)
87. (b) 88. (b) 89. l c! l 90_ , ... ,
83. (a) 84. (d) 85. (c) 86. (d)
81. (c) 82. (d)
96. (c) 97. (d) 98. (a) 99 . t a) 100. , .·\
93. (c) 94. (a) 95. (b)
91. (c) 92. (a)
101. (d) 102. (a)

EXERCISE Lf;:\.1,'E•r::..tl>
c/ 2y d ~' ~ J\ ...
I. The differential equation representing the family of (c) - 0 -4_,· = 0 t d l - , - _, - ~ _ 1 = I)
dr- ,L-,.-- J.t
circles touching y - axis at the origin is
4. If;P is an integrating fl.i-·tclr c, t'che Jit't~renti.tl ~uJ.Oc' n
(a) Linear and of first order
2.xv dr - l3.r2 - _1..: ) ,fr = 0, then the , .1l ue ,1r' :its
(b) Linear and of second order
(a.) - 4 tl>) 4
( c) Non linear and of first order
(c) - 1 l,iJ I
(d) Non linear and of second order 5. The parti.:ular integra l c'f the fotk,w ing Jifferenti.1I
· The general solution of the differential equation
2 eqnation
2 ~ '
2 ," f ? 1( 1- sr ~ -=- ..-v-.r+ [~,:,is ➔\ - 7 1 s in ➔. \ is
.I -. . ➔
ddx; =(d~ ) is
(a) x = c e-Y+ c eY (b) :x == c,eY + c2 C • ) .
(II) - t'
5 .~ -t- .,cl,--tI \•
1 2 (d 1 == c 1eY + c y
c) x = c ,e-Y + c 'I :x. f tecond order ➔
( 2
3. If e2x and xe2x are particular solut1o~s o a "th constant (h) 5 cos 4.r + 2 sin -lx
homogeneous differential equation WI
(d ~e-✓-' + ·2 ,:os ➔.\ + 5 si n ➔ \
coefficients, then the equation is )

2 d 2Y 5!!Y..+6Y "' O (d) 2 cos 4.r + 5 sin -ix


(a) d y - 4 dy + 4y == 0 (b) ~d:x2 - d:x
dx2 d:x
702
_________________
_________________
_ _ _ _ _ _ _:,:M~ATHEMAT1es

. 12. The general solution of the differential e q u a ~


6. If k is a constan t such that ~J' + k = ( x- 1)'
-, satisfies the
e •
dy = (l+ / )(e- x' -2x tan - 1 y ) is
differen tial equatio n x dy dx
= (x2 - x - I) y + (x - I), then
dx .> - I
k is equal to (a) e·' tan y =x + c
(a) I (b) 0 .l
(c) - 1
(b) e-., tan y = x + r:
(d) - 2
7. The solutio n y (x) of the differe ntial equatio ns (c) e-' tan y = x 2 + c
2
(d) e-x tan· ' y = x3 + c
d -
- ,y + 4dy
- + 4 y = 0 sat1s
. fy mg
' tI1e con d'.
dx· 1t1ons y (0) = 4,
dx 13. The solution of the differen tial equation -d 2y _ y == e'
dy dx 2
-(0) = 8 is
dx satisfyi ng y(O) = 0 a11d dy (0)
(a) 4e2x (b) (16x + 4)e- 2' dx
( c) 4e-2x + 16x (d) 4e- 2' + 16xe2 ' 3
8. The general solution of yy" - (y') 2 = O is is
2
2
(a) y = c 2e'C x
(a)y(x) =s1'nh x+-e
X X
(c) Y = (c2 -x) ec,x' (d) Y = C2e c,x
2

9. The solution of the differen tial equatio n X X


(b)y(x )=xco shx+
dy = / cosx+ cosy. Y(~)=o is
2e
dx xsin y -2y sin x' 2
(c) y(x) = s1'nh x - -e·
X r
(a) y2 cos x + x sin y = 0
2

(b)ysi nx + xcosy = ~ X
(d) y(x) = 2x cosh x - -ex
2 2
(c) y2 sinx + x siny= 0 14. An integrat ing factor of the differen tial equation 2.°C}· d:c
+ (y2 - x 2) dy = 0 is
(d) y COS X +X COS )I = -
7t
2
(a) y (b) _.!._
10. A particu lar integral of the differen tial equatio n y
d 2y
-- -
2
dx
16y = 4 sinh 2 2x is (c) _l (d) ~
)'2 y"
15. Ify = x cos xis a solution of an nth order linear differenti;1I
(a) .!_ (xe 4 x + xe-4x - 1) (b) .!_ (xe 4x - xe-4x + 1)
8 8 d"y d" - 1y I
. - -+a--+
equa t 10n dx" 1 dx" - t ... + a 11 - d, + , l,,_,- --L)
i \

(c) ¼(e 4
x -xe-4x +½) (d) ¼(xe x -e-4x
4
+½) with real constan t coeffici ents. then the kast possibk
1

] I. Lety 1(x) and y (x) be twice differen tiable function s on a value of n is


2
interva l / satisfy ing the differe ntial equatio ns {a) I (b) 2
(c) 3 (d) 4
dy1 - Y1 - y, =ex and 2dY1 + dYz -6y, = 0 . Then
dx 16. Let W[y 1 (x), y?(x)] is the Wronsk ian formed for the
- dx dx
yi(x) is solution s y (x) a~1d yz(x) of the differen tial equation /' +
1
a 1y' + a ,y = 0. If W '1' 0 for some x = x in [a, fi] thc:n
0
(a) Cie- 2x + c e3x - .!_ ex (b ) c1e2x + c2e3x +.!_ ex (a) It vanishe s for any x e [a. h]
2 4 4
(b) it does not vanish only at x = o
( c) ce2x +c e·3x _ _!_ ex (d) C1e - 2x +c, e3x +.!. ex (c) it does not vanish for any.,· E [a. fi l
I 2 4 - 4 (d) it does not vanish only at .,· = h
DIFFE RENTI AL EQUA TION

17. The general solution of y'(x + y) = y is 703


(a)x = cy+y 2 27. The solution of the differential equationyy
(b)x= cy -y2 ' + y2 -x = 0,
where c is a constant, is
(c)y = cx + x 2 (d)y =cx - x2 (a) y2 = x + ce-2x
18. The general solution of y' =
2x - Y is
(a) 2-x + 2 Y = C (b) 2-x - 2-y = C (b)y = x+ce -2x_.!_
(c)2x + 2Y=c (d)2 x-2Y =c 2
19. Solution of the differential equat
ion xy' + sin 2y =x3 sin 2 (c)y =
y is x +ce-2 x +.!.
(a)co t y= -x 3 +cx 2 (b)2 coty =x 3 2
+ 2cx 2 (d) y2 = x +I + ce-2'
-(c) tan y = - x 3 + cx 2 (d) 2 tan y
= x 3 + 2cx 2 28. The differential equation representing
all circles centred
20. A particular solution of the differ
ential equation at (1, 0) is
(D 4 + 2D 2 - 3)y = Ii' is
(a) (x + l )e' dy
(b) xi' (a) x+y - =I (b) x-ydy- =I
~
dx dx
(c) xex (d) xex
4 (c) y+x.:2'..=I dy -1
(d) y - x~
8 -
21. A particular solution of the differ dx
ential equation
y'" - 3y" + 3y' - y = e' cos 2x is 29. General solution of .:2'.. + 2xy =2e- x' is
dx
(a) _.!_ e" si n 2x '
(b) -1 e·' sm2x
8 8 (a)y= (2x+ c)e-x' (b)y= 2xe-x
(c) y = e-x (d) None of these
1 .
(c) - e' cos 2x . (d) e-' sin 2x 2
8 30. d ; + .:2'., _ 2y = 0, has the solution
22. ~et y~x). == x sin_x be one of
the solution of an nth order dx dx
lmea1 d1ffe~·e~1t1al equation with const (a) y = C,e- 2x + C2i'
ant coefficients.
Then the m1111mum valu e of n is (b) y= Ce-2x
(a) I (b) 2
(c) y = C e- 2' + C e-x + C
1 2 3
(c) 3 (d) None of the above
(d) 4 31. The differential equation y" +
23. The general so lution of the differ
ential equation 6y' + 9y = 50e2x have
y'" + y" - y' - y == 0 is particular integral
(a) (c 1 + xc2 + x2c3)e-' 2e2x
(a) - (b) 2e 2x
(b) (c + xc + x2c )e-x 3
1 2 3
(c) c e-' + (c + xc )e-x
1 2 3 (c) e2x (d) None of these
(d) (c, + XC2)e-' + C3e' 32. The particular solution of
24. The general solution of the differ
ential equation y"(x) -
4y'(x) + 8y(x) == l0e-' cos x is d3y .
-"f + y = COS (2x -1) IS
(a) e2' (k cos 2x + k sin 2x) + e-' (2
cos x + sin x) dx
2
(b) eh (k 1 cos 2x + ki sin 2x) + e' (2
1 cos x - sin x) 1
(c) e- 2' (k cos 2x + k sin 2x)- e'(2 (a) -[co s(2x -l)-8 )
2 cos x- sin x) 65
(d) e- 2' (k 1 cos 2x + k sin 2x) + e-"(2
1 2 cos x + sin x)
25. The general solution of the differential
equation (x + y- (b) _!_ [cos(2x -1)- 8 sin (2x -1))
3) dx - (2x + 2y + l)dy= 0 is 65
(a) In I 3x + 3y- 21 + 3x + 6y = k
1
(c) -[co
(b) In I 3x + 3y-2 l-3x -6y= k s(2x + 1)+8]
65
(c) 7 In I 3x + 3y- 2 I+ 3x + 6y =
k (d) None of the above
(d) 7 In I 3x + 3y- 21- 3x + 6y= k 33. The c-discriminant and p-discrimin
ant both contain
26. lf y(x) satisfies dy +2y =2+ (a) envelope (b) tac-locus once
e-x withy(O) = 0, then (c) node-locus
dx (d) None of these
34. The c-discriminant does not contain
one of the following.
Jim y(x) equals (a) The envelope (b) The tac-locus
x---+°'°

(a) 0
(b) I (c) The cusp-locus (d) The node-locus
(d) -1
(c) 2

,.
MAif.11:M

cosx)y" _ x ~.
equ ati on (si n x- x
70 4 41. General solution of · · ( srn .x)
integral of ,s
· en th at y = sm x a solution
<Pi(x) are particular · x)y = O, giv
+ (sm
35. Let <P, (x) and sta nts, then a Pl of y'
b being con
Uy )= eax - f(x ), a, in x
(a) C 1 sin x+ C 2 xs
0
L(y) = 2be x is
(b) <Pi(x)- b<Pz(x) (b) cl x sin X + C2 co s X
(a) bqi 1(x) + <Pk) 2
+ bq> z(x) (cf) b[<P 1(x) + <Pz(x)] (c) c, sin x + c2 x
(c) aq> 1(x)
ial n (a 2 - 2x y- y2)
different equatio
36. Solution of the (cl) C 1x + C2 sin x
dx = (x + y)2 dy is dN
2 2 2 - y3 = C
(a) 3a x - 3x y- 3xy y)d y = 0 an d -dM =_ ' th en
dx
42. If M( x, y)d x + N(x, dy
2 2 xy -y3 = C
(b) a x -3 y2 xz -3
(c) 3a x
2 - 3x y- 3x2y2 -y3 = C ex act
(a) the equ ati on is
ve
(d) None of the abo ( b) no t exa ct
n
sol utio n of the differential equatio
37. The (c) lin ear
(d) no n-l ine ar
(xsi,(1))dx- (ysi,(1J-+-Ois equation
dy + Py =Q is lin ear differential
43. The equ ati on dx
(a) cos ( ; ) =0
of first order, if
on s of x on ly
(a) P, Qa re fun cti
(b) sin ( ; ) = O on s of y on ly
(b) P, Qa re fun cti
on s of x an d y
(c) P, Qa re fun cti
x = constant
(c) cos (;)-log e
(cl) No ne of the ab ov =
fer n (D 4 + D2 + I) y
dif ent ial eq ua tio
= constant 44. 2.x
Th2e CF forsinthe

(cf) sin ( ; )- .log x ax - be~
en tia l eq ua tio n (a) e-< + e-x
ree of the dif fer
38. Or der and deg
4 .
sm
✓3 x )
, ( )2)11 are respectively (b) e 1 3
2
= p+(dp
d-p x12(c cos 2✓3 x+ C
d0 dB 2
(a) 2, I (b) I, 1
(c) 2, 2 (cf) 2, 4 X
✓3 + C4 sm. T✓3)
· ( cl cos 4x
+ e-x/2
2 uti on of thi s
2D + x 2
- X 2)y = (x - l)ex on e sol
39. (n x
is (c) ex ( C1 cos
✓3 x+C2 sm. ✓3)
differential equation 2
(a) x
(b) xeX 2
(cf) e1· + x
(c) e'"
dif fer en tia l eq ua tio n .
C sm
✓3 ✓3)x
sol uti on of the
40. The + e-x ( C3 cos 2 x + 4
2
1
+ y + fo
dy x-.c ( 2) 1 .IS e
-= --x+ 3
._ r )' -3 =-
y- 1 (cl) No ne of the ab ov
dx ation y' (y' +y)
of the dif fer ent ial equ
45. Ify' -x ct:. 0, a solution
ify(O) == O
+ y) - 1 == x(x + y) is given,
(a) y = x + log (x
3 (b) 1 -x + ex
(a) 1- x- e-·' . ,
(d) I + x + e"
y) + 2 4 (c) I + x + , -, equatro
(b) y =x- log (x + of the dif fer en tia l
3 6. Th e so lut ion

(c) y = x - log (x
+ y) + 2 fa
3 ,{,;ft+(:)'}+ y: - o
y) + I
(cf) 2y = x - log (x + (a) ax + by== C 2
2 ( b) ax + by = 0
(c) ax + by2 == I
(d) ax + by2 = 0
UIFFERENTIAL EQUATION

47. A particular ;nteg,a] of y'" - (a+ b)y' + aby - Q(x) ;,.


(a) eax f {e(a-b)x(f Qebx dx )} dx 705
54. The P.I. of the differential equation (IY - D)y = ~ + e-x,

(b) eax f {e<b-a)x(f Qe-bx dx)}dx D= !!_ is


dx

I .
(c) e-ax f {e<a-b)xf Qebx dx}dx (a) -(e·' +e-x)
2 (b) .!_x(ex +e-x)
2
(d) eax f e<a-b)x x{J Qebxdx}dx (c) .!_x2(ex +e-x) (d) .!_x2(ex -e-x)
48. 2 2
The homogeneo us differential equation M(x, y)dx, N(x,
55. Ifq>(x,y) = 0 is a singular solution, then q>(x,y) is a factor
y) dy == 0 can be reduced to a differential equation, In of
which the variable are separated, by the substitution
(a) p-discriminant only
(a) y == vx (b) xy = v
(b) c-discriminant only
(c)x+y== v (d)x-y=v
(c) Both p and c-discriminant
3 (d) None of the above
49. General solution of d y
dx3 2
d d 2
56. P.I. of --t +--1'. =x + 2x + 4 is
dx dx
- d2y dy
6 dx2 + 11 dx - 6 Y = 0 x2 x3
is (a) +4x (b) - +4
3 3
(a) Y =A~+ Be2x + Ce3x 3 3
(b)y=3~ (c) .::._+4x (d) .::_+4x 2
3 3
(c)y==A+Be2x 57. The complementary function of (Ir - a4 )y = 0 is
(d) None of these (a) y = c,ear + C2e-ax
(b) y = C 1e-ax + C ea:r + C cos ax.,. C sin ax
50· e-x (~ 1 cos ✓3x + Cz sin ✓3x) + C e2x is the general 2 3 4
3 (c) y = C1e-ax + C2ea:r + C3 sin ax - C cos a:c
solution of 4
(d) None of the above
(a) d3y!dx 3 + 4y = O
58. A differential equation of first order and first degree is
(b) d 3y!dx 3 - By= O homogeneous, if
(c) d3y!dx 3 +By= 0
(d) d3y!dx 3 - 2cf2y!dx2 + dyldx - 2 = O (a);={;) (b) drer. = constant
51. In linear ordinary differential equation, the dependent
variable and its differential coefficients are not multiplied dy
(c) dx = q>(x) td) None of these
together and occurs only in
(a) first degree 59. The integrating factor for the differential equation
(b) second degree
(c) third degree (d) fourth degree dy . ,
(X + I ) - - Y= e-'·' (X + 1)- is
52. General solution of the equation dr

d2 . dy . . = o 1
(a);,;i- (h)x + 1
(cosx-sin x) _y_+ 2 sm x--(cos x+sm x)y
dx2 dx
1
Given that y = sin x is a solution, is (d) x~ +1
(c) x~ + 1
(a) cl cos X + C'J! si.n X
(b) C sinx+C2 xsmx . . d\· tfr . . ...
1 60. Given, an equation - -·, =--'- and a SL'lunon ol 1t 1s y =
(c) C 1 sin x + C2 ~ ,L\·- dr
(d) C 1 sin x + C'J! 00
+ a sinh x + a2 cosh x, whert' a0 , a 1• , 1~ art' arbitra~
1
constants. then this solution is
d3z dzy 4 dy +4 = 0 has the solution
53. -+-+ - (a) particular solution
dx3 dx2 dx
'
(b) complete primitive
(a) y = C e-x + xC e-x + x2C3e-x (cl singular solution
I 2 . 2
(b) y = c, cos 2x + Cz s1\ ~ sin 2x (,/) None of these
(c) y = c,e-x + Cz cos 2X 3
(d) None of the above
706
2
d y dy · 67. . of -d y
Solution
2
2dy + ( I+--:;-2) y = xex is
- --
61. Solution of - + cot x-+ 4y cosec x- 0, 1s
2 _
dx 2 x dx x-
cb:2 d.x
i
(a) y = x(C 1cos x + C2 log x + - e-'')
2
(a)y= k1cos(2logtan%+k 2 ) (b) y = x(C 1 cos x + C 2 sin x)
(c)y= - x(C 1 cosx+C2 sinx)

(b) y = k1 sin (10g tan%- k 2 )


I
(d) y = x(C 1 cos x + C2 sin x + 2 e-')
. . . d2y
68. Solution of d1fferent1al equat10n - 2 + y = cosec x is
dx
(a) y = CI cos X + c2 sin X - X cos X + sin X log sin x
( b) y = CI sin X + c2 cos X - X sin X + sin X log cos x
(c) y = C 1 cos x + C 2
sin x-x sinx + cosx log sinx
(d) None of the above
dy y cos x + sin y + y
62. The solution of - + - ' - - - - - - - 0 is 69. The CF of x 2y" - xy' + 2y = x log x is
dx sin X + X cos y + X
(a) x[C 1cos (log x) + C2 sin (log x)]
(a) y sin x + (sin y + y)x = C (b) x[C 1 sin(log x) + C2 cos (log x)]
(b) y sin x + (sin x + x) = C (c) x[C 1 cos (log x)- C2 sin (log x)]
( b) y = sin x + y sin y + C (d) x[C 1 sin (log x)- C2 cos (log x)]
(d) None of these 70. Which of the following is not an integrating factor of
63. The nth order homogeneous linear differential equation xdy - ydx = O?
y<n> + PoY(n- I)+ .. . + P,l = 0 has general solution if the l 1
coefficients p 0(x), ... , p 11(x) on some interval I are (a) - (b) 2 2
x2 X +y
(a) continuous
(b) discontinuous (c) __!_ (d) ~
(c) discontinuous and differentiable xy y
(d) None of the above 71. For the differential equation .xy' - y = 0 which of the
following function is not an integrating factor.
2
64. Solution of x d y -(2.x- l) dy + (x- l)y = 0 is 1 (b) _1
cb:2 dx (a) -
(a) y = e-''(C 1 log x + C2 sin x)
x2 l
(b) y = e-'(C 1 log x + C2) (c) __!_ (d) _l_
(c) y = e-'(C 1 logx + C2 cos x) xy x+y
(d) y = -e-' (C 1 log x - C2 sin x) 72. The differential equation
65. The solution of the differential equation
(3a2x2 + by cos x) dx + (2 sin x - 4ay3)dy = Ois exact for
dy 3x 2 y4 +2.xy (a) a be any value, but b -ct: 2
- - ---,,.----,---,- is
dx - x2 -2x 3 y3 (b) a be any value, but b = 2
(c) b be any value, but a= 2
2
3 2 X
+-=-
X (d) a and b may be any value
(a) X )'
)' y 73. The solution of the differential equation
x2
(c) x 3 y2+-=C (d)Noneofthese (x 3 y3 + xy) dy = l is
y dx

66. A function f (r) = !!-_ { dx } satisfies the (a) -


1 =2 - y2 + AeY-' 12
dr l - cos t cos x X

differential equation (b) x=2-y2+Ae y2 12


(a) 1 +2.f(t)cost=0 (b) ! -2f(t)cos t=0 I ,
(c) -=2-/+Ae-y·l2
x ·
(c) df +2f(t)sinr=0 (d) df -2f(r)sint=0
dt dt (d) x=2+/+Ae- -"' 12
DIFFERENTIAL EQUATION

74. 707
If y 1 and y 2 are two solutions ofy" + p(x)y' + q(x)y = o,
then for general solution of this given equation, y and 1,
1 2,+ ~
y 2 are (a) y = Ae'+ Re h+Ce
(a) linearly dependent
,n
I,
(b) linearly independent
(h) y = A.-.-+ Re ·" + Ce > + ~
(c) proportional 10
1,
(d) None of these
(c) y = Ap' +Be.h +Ce-> + ~
10
75. The singular solution of ✓xdy + ✓Jdx == O is
; ,
1
(a) only x =0 (d) y = Ae'" + BP ' +c,, -: , _ ~
j(J
(b) only y =0
81. What is th e order and degree of a d iffs! rential equatio n
(c) both x = 0 and y = 0 1
2
(d) None of these J y
-l-+ I + --=- dv)
R¥J = fl '.'
d.x dx
76. 3 can be represented as
The equation y = Ae x + Be 5x
ta) First order. second degree
(a) y " - 8y + 15y = 0 (b) y" + 8y' = 0
(b) Fist order. first degree
(c) y' + 8y' = 0 (d) y" + 8y' + 15y = 0 (c) Second degree. fi m order
77. An integrating factor for y dx - x dy =0 (d) Second order. fim degr;;-e
82. Consider th e followin g d1fferenr ia l ct.ua.·w n,
(a)~ (b) Z 1. x2(v")6 +y-l } {l - 1_i - · ,.:; -
y X

+ d 2' 1( / )-w) =O
1 clc
(c) - 2- (d) x2 +y
2
X +y II. y' - 6x = {ay -b..t:1 ' / .!: -:c c1
Sum of order of I st di fferertu.l cq !.l:m,.m .1.0J
78. The solution of the differential equation x dy - y dx =
degree of second differeoti.1! cqUJii .:>o t:>.
2
Jx 2 + y dx is (a) 6 (f l -
(c) 8 \ ..i ..J
2
(a) Y + J x + y2 = xc ..1 - ,
Solution of di ffc-renr i.:iJ c-4 u.:irk,n --, - ·! \ -= ~C1.: l"ll' b
83.
2 2 2 ..!\
(b) y +Jx +y =x c

2 2 T t.. .. ~ ~·L,~ , L\ - .) !ll 1t1 Iv~ ..:0::. ..u -


(a) y ::::: c·, jin ,n 12
(c) y-Jx +/=xc

2 3 .\
(d) y -Jx +/=cx - sin m
II
79. Particular integral of the following differential equation
e 2 x log x . . . . (n) _I ~ ( ·1 ,,,s / Lf r t .2 ~ lll fL \ - --, .:us 'lt l0g S lll ,1.f -
(IY - 4D + 4)y = - 1s obtatn by mtegrat111g ,,
X

2
l '\.l.S ll \
(b)~ II
(a ) ~ 2
2
(d y t · .:us IL\ T l·! s in ,u· r --, ,:l'S fL \ I..~ ,<-\S ILI •
1
"
(c) (log/' (d) ( ~ ) '
- 1·111 fLf
II
The solution of the differential equation
80. l d ) No n,· ,, r" 1ht' ab,,, <'

3X
d3y d 2y dy
- -z--5-=6 y=e IS
dx 3 dx 2 dx
708 MATH EMAlies

84. General solution of differential equation 90. The nodes have a double point
(a) with distinct tangent
dy (b) with coincident tangent
dx + tan x tan y =cos x sec y is (c) same tangent (d) None of the above
(a)siny=(x+c)cosx (b)siny=(x+c)siny 91. The solution of the equation y' sin x = y logy satisfying
(c)sinx=(x+c)siny (d)cosy=(x+c)sinx
85. The solution of the differential equation the initial condition y( ~) =e is
2
dy2 dy . (b) ecot x/2
x --4x-+6y=x IS (a) etanx/2
2 dx dx

X (c) log tan(%) (d) logcot(%)


(a) y = c 1x + ciX2 + -
2
92. For a given differential equation
x2 (a) an envelope gives a singular solution
(b)y=clx + cr 3 +- (b) node-locus givens a solution
2
(c) cusp-locus gives a solution
(c) y = CJX2 + Cr3 + 2X (d) (a), (b) and (c) are false
93. An integrating factor for (cosy sin 2x)
x2
dx + (cos 2 y- cos 2 x)dy = 0 is
(d) y 3
= c Ix 2 + c-x
r + - (a) sec2 y +secy tan y
2 (b) tan 2 y +secy tan y
(c) I/(sec 2 y+secytany)
86. Let y be a function of x satisfying dy = 2x3 -Jy - 4.xy of
dx (d) I/(tan 2 y +secy tan y)
y(O) = 0 theny(l) is 94. The p-discriminant does not contain one of the following.
(a) The envelope (b) The tac-locus
(a) _I_ (b) _!_ (c) The cusp-locus (d) the node-locus
4e 2 e 95. If Mdx + Ndy = 0 is differential equation and is
(c) el/2 (d) e312
__!_(dN _dM) is a function ofy alone, sayf(y), then
d2 d M dX dy
87. Let y = eX be a solution of x-1'.. _ -1'.. + (I- x)y = O.
dx 2 dx
integrating factor is
Then, the second linearly independent solution of this (a)eX (b)eX+y
ODE is
(c) eJ f(y)dy (d) None ofthese
-x I
(a) xe +- (b)
2
½( x-½ }-x 96. In equation Mdx + Ndy = 0 has the formft(x, y)y dr +
fz(x, y)xdy = 0 and Mx - Ny= 0 an integrating factor is
(c ) - -I ( x+- 1
1 ) e- x (d) xe -x - - (a)Mx~Ny \ (b)My-Nx
2 2 2
88. The integrating factor of the DE (2xy 4eY + 2xy3 + y) dx + (c) - - - (d) None of these
Mx-Ny
(x 2y4eY-x2y2- 3x)dy = 0
97. Any equation contai~s n-arbitrary constants, then the
2
I order of differential equation derived from it is
(a) 4 (b) L
X x4 (a)n (b)n-1
I (c) 2 (d) n + l -'r
(c) -;( (d) None of these 98. The differential equation, derived from y = .4e2' + Be -
y have the order, where A, Bare constants
89. A particular solution of (a)3 (b)2
2 (c) 1 (d) None of these
2 d y dy y I .
X -+2x-+-=- JS
2 2
dx dx 4 ✓x, 99. If eax u(x) is a PI of d y _ 2a dy + 0 2 y = f(x), where a
I dx 2 dx
(b) logx
(a) 2✓x 2✓ x 2
· d u
JS a constant, then - is equal to
(logx)2 dx2
(d) (logx) ✓x
(c) 2✓x, 2 (a) f(x) (b)f(x) eax
(c)f(x) e - ax (d)f(x) (eax + e--ax)
709
DIFFER ENTIAL EQUAT ION

The solutio n of the differential equati on (2ax


+ by) y dx = (x + y) is
I 00. The solutio n of the equati on (x + y - 2) dy
dy = 0 is 107. dx
+ (ax + 2by)x
(a) axy2 + bxy = c (b) ax2y2 + bxy = c = log(y -x + 1) + C
(a) (y+x)
(c) ayx2 + bxy2 = c (d) axy + by2 = c (b) (y-x) = log (x + y- I)+ C
sed as
101. The equati on x = A cos (pt - a) can be expres (c) y- 2x = log (x + y- 1) + C
d 2x (d) y + 2x = log (x + y +I)+ C
(a) - =x (b) d2x - ns no
dt 2 - - p2x A solutio n of a differential equati on which contai
dt 2 I 08.
arbitra ry consta nts is
(a) particu lar solutio n (b) genera l solutio
n
(c) d2x =0 (d) None of these
dt 2 (c) primit ive (d) None of these
2 -xy)d y ion,
102. Integr ating factor of the DE (y2 + 2x y)dx + (2x3 109. The soluti on of the differ ential equat
= 0 is dy .
d2y dy x = 0 1s
- - - 3 - + 2y = ex y = 3 and -dx = 3, when
3/2 dx2 dx '
(a) xll2y-5l2 (b) ~
YS/2 -xeX
(a) y = 2eX + e-2x -xe-x (b) y = 2eX + e2x
(d) x-s12ys12 (c) y (d) y = eX- e2x + xe"
= eX + 2e2x -x2eX
(c) x - 512y-112
110. Order and degree of the differe ntial equati on eY"'-x y" +
. d 2y dy y = 0 are respec tively
103. Gener al solut10n of 2 - -2 + 5 -dx + 2 y = 0, is
dx (a) 3, 1
I (b) 3, not define d
(a) Y = Ae-2x (b)y = Be 2x ( c) not define d, I

I
(d) both order and degree are not define d
neous
(c) y = Ae-2x + Be -2x (d) None of these 111. Consid er a genera l first order linear non-h omoge
equati on withy = cp(x) as its solutio n.
es nodal ordinary differential
104. The c-disc rimina nt when equate d to zero includ eneou s
Let \jf(x) be the solutio n of the associ ated homog
locus genera l
equati on and C an arbitra ry consta nt. Then, the
(a) once (b) twice equati on has the follow ing solutio n.
(c) thrice (d) None of these (b) y = Ccp(x) + \jf(x)
(a) y = C<p(x)
(D 2 -D+
105. Solve (D + l)x + (D- l)y= e', (D2 + D + l)x+ (c) y = C\jf(x) + <p(x) (d) y = C\jf(x)
d ntial
=- 112. Which one of the follow ing is the solutio n of differe
l)y = t 2 where D
dt equati on?
(a) Envel ope (b) Cusp- locus
1 2 1
(a) y = -(3t + t - 2e ) (c) Node- locus (d) Tac-lo cus
3
113. The soluti on of the differ ential equat ion y
I 3
(b) y = -(2t +t -3e1) px+ ✓4+p 2 is
4 2
2 2 =4 (b) (y-Cx ) 2 -4C =0
(a) (y-Cx ) -C
2 ' 2 + C2 = 0 (d) (y- Cx) 2 + 4C2 = O
(c) y = -I (2t + t - 2e) (c) (y- Cx)
2 .t2 (3a 2 + by cos x) dx + (2 sin x
II 4. The differe ntial equati on
2 I
- 4ay') dy = 0 is exact for
J
(d)y= -(2t+ t -3e) (a) a= 3, b = 2 (b) a= 2, b = 3
2
primit ive of (c) b = 3, b = 4 (d) a= 2, b = 5
I 06. The numbe r of arbitra ry consta nts compl ete
The orthog onal traject ory to the family of circles
.t2 + y 2
I 15.
'ffi . I . dsy d4y . s
O contam differe ntial equati on,
-
d1 erent1a equat10n - 5 + 2 4
= = 2cx (c arbitrary) is descri be by the
dx dx 2 2 y' = 2ty
(a) (x + y2) y' = 2xy (b) (x -y2)
(b) 4 (c) (y2 -x 2 )y' = xy (d) (y2 -x2) y' = hy
(a) 5
(d) None of these
(c) I
710
-
116. Which of the following is/are integrating factor of the
120. The solution of differentia l equation c o n ~
M,~~~r
differentia l equation xdx + ydy = 0
as arbitrary constant as the order of a differ rn~y
1
equation, then the solution is said to be enttal
l l
(a) x2 + y2 (b) (x2 + y2)2 (a) particular solution
(b) complete primitive
l (c) singular solution
(c) - -2 - 2)" (d) All of these
(x + y (d) None of these
121. The nth order ordinary linear homogene ous differential
117. The solution of (x + 1) dy equation have
=1- e-y is
dx (a) n-singular solutions
(a) (x+ l)e>'=x + C (b) (x + 1)(1 + e>') =C (b) no singular solution
(c) one singular solution
x+l =C x+l =C
(c) (l+eY) (d) (1- eY) (d) None of these
122. The differentia l equation whose linearly independent
solutions are cos 2x, sin 2x and e-x is
118. The differentia l equation dy =k(a- y) (b- y) when
(a) (IY + D2 + 4D)y = 0
dx
solved with the condition y(O) = 0, yields the result (b) (D 3 -D2 +4D-4)y =O
(c) (D3 + D2 - 4D - 4)y = 0
b(a-y) (a-b)kx
(a) a(b-y)= e (d) (D3 -D2 - 4D + 4)y = 0

123. The differentia l equation : +Py= Q, P and Q are


a(b - y) (a -b ) kx
(b) b(a- y) =e
functions of x only, have the integrating factor

(c) b(a-x) =/b-a)k (a) ef Pdx (b) el Qdy


a(b-x)
(c) ef Pdy (d) ef Qdt
(d) xy = kC
119. 124. Ify 1(x),yz(x) are solutions ofy" + xy' + (1 -:r)y = sinx.
y = px - 2p 2 find its singular solution
then which of the following is also its solution?
(a)8y=x2 (b)x 2 -4y=O
(a) Y,(x) +yz(x) (b) y 1(x)- YN)
(c) x 2 = 2y (d)x 2 =y
(c) 2y 1(x)- yz(x) (d) y 1(x) - 2yix)

Answe r Sheet - 11
1. (c) 2. (c) 3. (a) 4. (a) 5. (c) 6. (a) 7. (b) 8. (d) 9. (b) to. (b)
11. (c) 12. (a) 13. (a) 14. (c) 15. (d) 16. (c) 17.(a) 18. (d) 19. (a) 20. (dl
21. (a) 22. (d) 23. (c) 24. (b) 25. (d) 26. (b) 27. (b) 28. (a) 29. (a) 30. la)
31. (b) 32. (b) 33. (a) 34. (b) 35. (d) 36. (a) 37. (c) 38. (d) 39. lb) 40. la)
41. (d) 42. (a) 43. (a) 44. (b) 45. (a) 46. (c) 47. (b) 48. (a) 49. (a) 50. lb)
51. (a) 52. (c) 53. (c) 54. (b) 55. (c) 56. (c) 57. (b) 60. lb)
58. (a) 59. l<l)
61. (a) 62. (a) 63. (a) 64. (b) 65. (c) 66. (c) 67. (d) 70. (dl
68. la) 69. (a)
71. (d) 72. (b) 73. (c) 74. (b) 75. (d) 76. (a) 77. (c) 80. l•·)
78. (b) 79. (c)
81. (d) 82. (d) 83. (c) 84. (a) 85. (c) 86. (a) 87. (c) 90. (a)
88. (c) 89. (c)
91. (a) 92. (a) 93. (c) 94. (d) 95. (c) 96. (c) 97. (a) 100. le)
98. (b) 99. lC)
101. (b) 102. (c) 103. (c) 104. (b) 105. (cl) 106. (a) 107. (b) no. lb)
108. (a) 109. (b)
111. (c) 112. (a) 113. (a) 114. (a) 115. (d) 116. (b) 117.(c) 120. lh)
118. (a) ll 9. l<1)
121. (b) 122. (b) 123. (a) 124. (c)
71 1
EQ UA TIO N
DIF FE RE NT IAL
LE VE L - I
S O LU TI O N S
H IN TS A N D ~ (l
r x < 0 = x fo r x
6. Here, I x I = - x fo 3 fo r x ~ 0
dy an d So , g( x) = x
1. we ha ve dx = 2y + z = -x3 fo r x < 0
... (i) 3
f(x ) x =
an d s
dz l eq ua tio n is ho mo ge ne ou
... (ii ) fer en tia
dx =3 y 7. The given dif
I 1
we ge t -
fro m 3 (i) + (ii ), So IF = - -- = -- Z
Ny x2 sin
' . . M x+
dy dz X
- = 9y + 3z
3 - +dx
dx tio ns by I.F.
dif fer en tia l eq ua
M ult ipl yin g the
. 1" d = 3( 3y + z)
im p tes dx (3 y + z) y) dx +1-- co t-y dy
y o t- =O
X
1 -c
-- 2 X X
ge t ( X x
on in te gr at in g, we
d( 3y +z ) =f 3d x
f 3y + z f Mdx =JU-; co ti )d x
c
im pl ie s ln (3 y + z) = 3x + ... (i)
3 i)
So, 3y + z = ke x s = lo gx +l og si n(
n is homogeneou
fer en tia l eq ua tio
2. Si nc e, gi ve n dif
gs in 1'. . .. (ii )
2 =f_! _c ot Zd y = lo X
-3 x 2y + x 2y = - 2x y Ndy X
So , I.F. = M x+ Ny I X
on ter ms on ce
) wr iti ng co mm
. Ad din g (i) an d (ii
aM aN . sa tis
1s fie d (ex ac t)
3. He re --: ;- = --
ax
oy log x + log sin 1'.
=c
X
dx 2x near)
2 - + - y = 3 (li . y ,
dy xs 1n - =c
im pli es X
2y ·
dy
dx = Jy - 2 x (h om og en eo us )
dy + (3 + _!_) y = e-2 ,·
8. we ha ve x
dx
we ha ve Z. dy = -[-x_z_+...::y_z___l_O]
4. Xz + yz +5 f (,+ .!.),LY
X dx x = xe 3-'
x =0 I. F. = e
+ 5) yd y + (x2 + y2 - IO)_xd
im pl ies (x2 + y2 y) + 5y dy - 10xdx = 0 log x, Th en
(xdx + yd 9. Le t= =
im pl ies (x2 + y2)
z 2 d d
. (x z + y2)2 5y - 5x = c -= D i
. -'- --- -=-- .:.- + - - 2 - = Xd,·
1m ph es 4 dz
2
2 + I Oy2dy - 20 x2
= c
im pl ies x + y2)2 4 x2 + 1Oy - 11 = 0 , d2
2 y - 20 x- - =D 1(D 1 - 1)
So , x 4 + 2x y2 + dy = 0 dx 2
)d x - xy si,1 (y )
5. we ha ve 2c os (y 3 s to
x eq ua tio n ch an ge
M ul tip ly in g by 4y sin (y ) dy = 0 Th e differential
2x3 co s (y ) dx - x 2 d2 y
dy
aM -5 x- +9v =o
4x3y sin (y ) x -; tit .
ii; " = - dx -
+ 9) y = 0
im pli es (D 1(D 1 - 1) - 5D 1
3 (y ) y =O
aN = - 4x y sin (D r - 6D 1 + 9)
an d ax 2
(D 1 - 3) y '= 0
3= '= (c + c lo g x) x 3
aM = aN y '=(c, + c 2:) x 1 2
So,
Si nc e, ay ax
in teg ra tin g fac tor.
So, x3 is
"

712 MATHEMATICS
~
=-{D+ l)cos(2 x+3)
I 0. Let log x = z, Then
= 2 sin (2x + 3) - cos (2x + 3)
2
d d 2 d
- = x-=D, , x - 2 =D1(D1 -1) d d 2
dz dx dx 14. Let z = logx, Then x-=- =D1 ,x D2
dx dz
The differential equation changes to
2 d2 dz d
(D 1 (D 1 -J)-5k D 1 +1)y= 0 =x -=---
2
=D1 (D-I)
dx dz 2 dz
(D/ + (k-1)D 1 + l)y=0
So, The different ial equation becomes
Solution is y = (c 1 + c2z)et
(4D (D - 1)- 8D 1 + 9)y = 0
1 1
So,D/ + (k- l) D 1 + l)y = 0
(4Df- 12D1 + 9)y = 0
implies k = -I
(2DI - 3) 2 y =0
11. Yix) and y 4(x) are linearly depende nt if yix) = kJlix)
where k is any constant So, y = (c 1 +'½log x) x½
ay 1(x) + by2 (x) =k(cy 1(x) + dy2 (x)) 16. g(x, y)d.x + (x + y) dy = 0 is exact if
(a - kc)y 1 (x) + (b - kd)y2(x) = 0 dM dN
Since, y 1(x) and y 2(x) are independent dy ax
So, a-kc= 0
and b-kd= 0 implies dg = l
ay
or !!_ =!:._=k g(x, 0) = x 2
C d
So, g(x, y) = x 2 + y
So,yix) andyix) are independ ent if 2
(x + y)dx
I Md.x = f y=const
a b
-*-
c d x3

implies ad* be = 3+xy

12. we have dy - 2x = <P(x) JNdy = f(x+ y)dy


dx
y2
or, dy = 2x + <l>(x) = xy+T
dx
So, general solution is
2
Thus, y = x + <l>(x) dx + c
J x3 y2
implies y = x 2 + c for x $ 0, 3+xy +2=c
but y(0) = 0 implies c = 0 implies2x3 + 6xy + 3y2 = c
So, y =x 2 for x $ 0 (D 3 -3D2 + 4)y = O
17. we have
y = x 2 + x for x > 0 (D 3 + D2 - 4D2 + 4)y = O
2 + D + 3)y = 5 cos(2x + 3)
implies
13. we have (D
implies (D + l)(D2- 4D + 4)y = 0
1 implies (D + I) (D- 2) 2 = O
So, P.I.- 2
5cos(2x +3)
(D +D+3) So, y = c Ie-x + c2e2x + c3xe 2'
1 18. we have , dy
(-4+D+ 3) 5cos(2x +3) l- y = -d.x

= (D+l)
(D2 - 1) 5 cos (2x + 3) implies
c~l -t )dy =-dx
DIFFE RENT IAL EQUA TION 713
y-J x2
- 2x.Y.Y + I + y2 = 0
impl ies I n - - =-x +c implies-
y
Orthogonal trajectories are given by
dx
impl ies 1'...=--!
y = cle-x - x 2 + y2 + I + 2xy -dy = 0

21. we have (D 3 - 9D)y = cos x


I implies D(D2 - 9)y = cos x
implies y===-
1-c, e-x C.F. = C 1e3 x + C 2 e -3.r + C3

l l l
Hen ce, Iim y =I COS X =- - - COS X
X➔ -
and P.r. 2 JO D
D(D -9)

dy So, complete solut ion is


19. we have dx =ay -by2 .
l
3 SID X
3
Y(x) =C I e x + C2e - Jx
+ C - -J 0
_!__ dy I
implies y2 dx - a . -y = -b 22. we have (D 2 + 6D + 25)y = 0
?. = -3 ± 4i
So, y = e - Jx [A cos 4x + B sin 4x]
I dy
Let
_!_ _ z, thedzn-= --·
- - but y(O) = 0 impli es A = 0
y y 2 dx
dx
Hence, y = Be-3x sin 4x
So, Equa tion becomes, 23. weh ave( D2+ 4)=0
So, solut ions is y = c 1 Sin 2r - c2 COS 2 :r
dz
dx+ az= b but y(O) = I impli es c2 = l
and y'(O) = 2 impli es 1c 1
Henc e I.F. = eax
= 2 implies c ! = I
+c
f Hence y = sin 2T + cos 1x
So, Solu tion is z · e= = b • e= dx
24. we have xdy = (y + xe-Y x)dT
= !!_ ,e= +c implies xdy - ydx = x e--' -" dx
a = dx
e-'ix( xdy- ydT )
implies 2 X
b -ax X
= -+c e
impl ies y a impli es d(e-1' -<) = dtln x)
implies 11,· x = In x + c
but y(O) = Yo
27. Ques tion shou ld be
L.
.r~ = Jx3 + x +
C = (_.!.__!!_)
impl ies Yo a Diffe renti ating w.r.t. x , we get

dY '
1,·-' - = 9r +I
_!_ = !!_ +(_.!._ - !!_ 1-ax . di:
Henc e y a Yo a I
dv cir of
Repl acing -'- bv - - , the equa tion
a ,fr dv
Asx ~ oo , y= -b .. . (i) ortho gona l trajec tory is given by
lO. we have (x - J )2 + y2 + 2ax = O r
ci. ,
+ I
Diffe renti ating w.r.t. x, we get -
-2v. -,~v = 9-r

2(x - 1) + 2y y' + 2a = 0
impl ies 2a = -2(x - 1) - 2yy' 2dx dv
a = - (x - 1) - yy' impli es 1 y
9x + 1
get
Putti ng expr essio n of a in eq. (i), we 0 Integ ratin g both sides . we gel
(x - 1)2 + y2- 2x((x - ]) + yy') =
0
impl ies x2 _ 2x + 1 + y2 -2x2 + 2x + 2xyy ' =
1
714
~ , \llcs
Now, f Md.x .· J< x2y+xy2 )dx
-2 tan - 1 .Jx = - ln(y) + c
3
3
implies2 tan- 1 3x + 3 In Iy I = k x y x 2 y1
= -+-
3 2
28. -
"JM = -"JN 1s
. sat1s
. fi1e d , So, equation
. 1s. exact
1
dy dX 2
and f Ndy = f (~+ x y+s in y }y
1 - ,2
- xe -' + c ... (i)
JMd-,,; = 2x-
x 3 y x 2 y1
= - 3- + -2- - cos y
f Ndy = f 2x ye-i dy ...(ii)
So, solution of differential equation
= - xe-i + c
x3y x2/
Adding (i) and (ii), writing the common tenn once. - - +- - - COS y =C
3 2
2x 3 -xe- y' = c
33. Multiplying the differential equation by J__. we get
x2
implies x(2x 2 -e- y' ) = c

dy
by2) cy
2
29. we have X dx =y (2 +x - dx+-dy = 0 X

It is exact

implies = aM oN
f dy)' f dxX So, ---
dy ax
implies y= ex
2by = cy
but y(l) = 1 implies c = 1 implies 2
xz X
So, y=x 2b + c = 0
30. we have (D - 3)y 1- 4y2 = 0 35. we have y = In sin (x + a)-"- b
4y2 + (3 - D)y 1 = 0 implies y' = cot l-'" - a)
Solving equations, we get implies y" = - cos~--.c l-'" - a)
(D2 - 6D- 7)y 1 = 0 = - [! - CO[~ p· - a)]
So, y 1 =c 1e7x+c 2e-x = -{ l - _,,.:)
31. e + .XJ1 + X sin J' + eY = C.
36. we have J (y + 2) dv = J sin .r dr:
Taking total differential
(e + y + siny)dx +(x + x cosy + eY)dy = 0
( \" + 2) c = - Cl)S X +C
implies )
32. we have
At, x = 0, y = O implies c = 3
(x2y + xy2)dx + ( ~ + x 2 y + sin y }y = 0 So, lv + ::!)' = - ::! cos x + 6
Hence. _v(y + -n = ::!l I - cos x )
aM 37, Here y = c I sin x + c 2 cos x
- = x 2 +2xy
ay .vl 0) --= I implies c·, = I
y( rr) = I implies c 1 =- l
and oN = x2 + 2xy
ax - I -, t' ' ~
· 1· 1 . ,
I .t --= t' -
I .
SIii X
38. PI. . = -
(D - 1) - D2
aM - dN
Since,
ay- ax
by using (- ' - v = t"" __ I_\')
So, differential equation is exact _((I)) flD +a )
DIFFERENTIAL t:QUATION

= - eX sin x 715
Now,y 1 (x cos x - sin x)-y cosx
2 are two linearly independent solutions of
= ex(x cos x - sin x) _ xeX cos x given differential equation
=- eX sin x = P.I.
40. (x - h)2 + (y- k)2 = a2
y1 Yi II e-" xe-"
where h, k are parameters .. . (i) Now, W(y i, Y2 ) = , = x x x II
IY'1 Y2 e xe + e
Differentiating w.r.t. x, we get
= x e2x + eh - x e2x = e2x
(x _ h) + (y- k)y' = 0
42. Given, differential equation is
Differentiating w.r. t. x again 1 + (y - k) ... (ii)
yll +yl2=0 I
y = px + -
p

implies 1 + y,2 implies p 2x - py + I = 0


(y- k) = ---;-
y .. .(iii) So, p-discriminant = 0
From (ii), we get (x- h) = -(y-k)y' implies(- y)2- 4 · x · I = 0
implies y2 = 4x
y'(l+ y 2)
43. Given, differential equation is
y" .. . (iv)
(y - c)2 = x (x - a )2
Putting (x - h) and (y- k) in (i), we get implies y2 + c2 - 2cy = x(x - a)2-
impliesc2 - 2cy + y2 - x (x - a )2 = 0
y ,2. (I+ y ,2.)2 + (I+ y ,2)2 = 2
0 So, c-discriminant = 0
y "2 y"2
implies4y2 - 4[y2- x(x - a)2] = 0
implies ( 1 + y'2 ) 3 = a2y''2 implies 4x(x - a) 2 = 0
41. A.E. for given differential equation is implies x(x - a)2 = 0
m 2 -2m + 1 = 0 44. Wehave
(m - 1)2 = 0 ⇒ m = l, 1
- IyY,' (0) Y2 (0)
W(ypY2) - (0)
implies Ye/= (C1 + xC2 )~ y '2 (0)
I=IO1 II I = - I
1

Now, Y1 = ~,Y2 =x~n

. ..... ,.._.... _ . ,.r:·i.:.·J,>.~"•"-'.


H, Nrs ANoisoi.u:rr,,arsfEost sui HT;(" _,.-:- <?:....· ,_~-"-•:c,o..;:s:",;d;:_:..:;;i-. :;.~-:-J,.•.-- -'.'ii;_.; ,._: • -.,:

implies y = - log (x - c) - c 1
1. Equation of circles touching y axis is given by 1
implies x + c = c/e-,
(x - a 2) + y2 = a 2 where a is parameter implies x = c 1' <'.,, , c' ,
implies x 2 + y2 - 2ax = 0 where c 1' is constant. -
Differentiating w.r.t. x, we get 4. 2xydx - 3x2 - y2 )dy = 0
Let us multiply it by y •, we get
2x + 2yy' - 2a = 0
implies 2,:v" + 1 ,fr - (Jx.:-,1 · 1 - _1.., T .:-)a'.v = o
implies a = x + Y.Y'
c)!vf cJN
So, differential equation will be - = -
So, d_\' dX
x2 + y2 - 2x(x + yy1) = 0
implies 2( a + I) ,,y ' == -6.'ly•'
· - dy 2 2 - o a + l =- 3
imp 11es 2xy - + x - Y - implies
dx . . r1near and of first order implies
a = - -L
So , differential equation 1s non- ,, _ y'2
- I ii'
- t! -
~ ~hDe y- 5. P.I. - (D 2 +2D+5) [54 + l8c<>s>,
,. - 7 1sio>x]

J? == f dx'
on integrating, we get 5 1 ~~+ __I_ _
2
- 4(D 2 +2D+5) (D +"!. D + S)
__!_ == X + C
[18 cos 4x - 7 1 sin 4x]
y',
implies ___ 1
y - x+c
implies
MATHEMA7
1
71 6 ~
- 8. we have yy,, -y,2 - 0
5 1 x/2 1
-o-- --~e + - - - y" = 2'...
¾ ½
4 ( + 2 . + 5) (-16 + 2D + 5) implies y
y'

(18 cos 4x - 71 sin 4x) dy' d


on integrating, we get f y' =f :
1 x12 (2D + 11) .
=-·e + (18cos 4x-7ls m4x) lny'= lny+ c
5 (4D 2 -121) implies
implies y'=cly

= ..!_ · ex/2 - _!_ [-925 sin 4x - 370 cos 4x]


185 Again on integrating, C1dx+k
5 I dyy = I
implies lny=c 1x + k
= ..!.. ex/2 + 2cos 4x + 5 sin 4x
5
implies y = c2ec,x
6. we have -dy - ( x -1 - - y =1- - I) I 9. we have (y2 sin x + x cosy) dx - (x sin y - 2y sin x)dy"'
dx X X 0
Here M = y2 cos X + cosy
l))dx -(x-1)2
X
If= eJ(.!.-(x+ -2- N= -(x siny-2 y sinx)
So, • • = xe and
aM .
-(x-1)2 So, ~ =2yc osx-s mx
-2~
Thus, solution is yxe
dN .
and dx =-ysm y+ 2ycosx
I) (x-1)2
·xe_ 2 _ +c
= f (1-; rJM dN
Since,
rJy = ax
-(x-1)2
So, differential equation is exact
= f (x-1) e --dx+ c 2

Thus f y=const
Mdx = f (y 2 cos x + cos y )dx
_(x-1) 2
2 +c = y2 sin X + X COl> y
= -e
and Ndy = f(2y sin x -x sin y)dy
(x-1)2 I x=const

implies xy = -1 + ce -2-
= y2sinx +xcos y
(x-1)2 So, solution is given as
implies xy+ 1 = ce 2 y2 sin X + X cosy = C
So, k= 1 7t
At x = ~, y = 0 implies c=-
7. we have (D2 + 4D + 4)y = 0 2 2

7t
implies D+2)2 y=O implie s y2 sin x + x cos Y =
2
So, solution is y = (c 1 + C-;X) e-2x
but y(O) = 4 implie s c 1 = 4 d2
10. we have -f - l 6y = 4 sinh2 2x = e4x + e-4x - 2
dx
So, e-2x + c e-2x
2
dx r
dy =-2(c I + C-x)
C.F. = c 1e4x + c 2e-4x
1
dy . 1· and P.1. (D2 -16) (e4x + e-4x - 2)
Atx=O -dx =81m p1esc2 =16
'
So, solution is given by (16x + 4) e-2x 0
1 e4x + 1 e-4x _ ____!__2e
(D-4)( D+4) (D-4)( D+4) D2 -16

I
I
I
I

_
ATIO N
71 7
DIFF ERE NTIA L EQU

--4 .r +_J_ ex
= __1_ _J_e4x +~ (_J _)e 8 So, y = c 1 sinh x + c 2 cosh x + ~2
) 8
(D +4
(D -4) 8
O,
but y(O) = 0 imp lies c 2 =
--4r ]
1[ 1 4x I +1
= 8 D - 4 e - D+4e . y
Thus = c 1 sinh x + ~2 ex

= _J_ [xe 4 x - xe-4x + I] dy I


+xex)
8 implies dx =c 1cos hx+ 2 (ex
dy ,
11. we hav e dx - Y1 - Y2 = eX x= O dy =2
At 'dx 2
implies (D -l) y 1 -y2 = eX .. .(i)
3 I . .
dy1 dyz =c ,+ 1 s
1m p1e c 1 =1
2-+- dx
- 6 yI =o 2 2
dx
.. . (ii)
implies (2D - 6)y 1 + Dy2 = 0 Y = sinh x + ~ ex
x, we get So, 2
Dif fer ent iati ng (i) w.r.t. ... (iii)
D( D- l)y1 -D y2 =eX
get I
Ad din g (ii) and (iii) , we 14. Here I.F. = 2 , we get
2 6)y = e' y
(D + D - 1
C.F. = c 1e2x + c2e-3x
equation by -;..
1 ex=-_!_ex Multiplying differential y
and P.I. = 2 4
(D +D -6)
is given as 2
So, com ple te sol utio n 2x x+ [ I -x-) dy
-d =0
y y2
y = c,e zx + Cze3x - _!_ ex
4 -2x
<JM -2x <JN
2 -I ) Now, a; -= 7· ax = 7
dy 2 ( e -x - 2x tan y
12. He re, - = (1 + y )
dx dN
<JM
dy 2 Since,
e-x - 2x ran - 1 y ay = ax
imp lies 2
1+ y dx ct
So, equation becomes exa
x2 15. y =x cos x
. . I -+dy 2 x tan -I y = e- y' = - x sin x + cos x
imp 11es - - 2 implies
l+ y dx y" = - sin x - x cos x
- sin x
implies .. . (i)
Let tan - 1y = z implies y" + y = -2 sin x
implies y"' + y ' = -2 cos x
dz +2 xz = e-x, sin x . . . (ii)
Then, implies y" "+ y" = 2
dx (ii) giv es
..., No w (i) +
I.F. = e y"" + 2y" + y =0
So, n =4
So,
Thu s , sol utio n is
x2 x2 J dx + c 11. Here, y'(x + y2) =y
,
ex- . z = e . e-
dx
= x+ c implies Y dy -x = y2
2
imp lies ex tan - J y = x + c
dx I
+ c2 cos h x implies - - - · x =y
13. He re, C.F. = c 1 sinh x dy y
I x
1 x
and Pl = -- -e -- -- -- e
(D -l) (D +l) I __!_ d)'
.. (D 2 -1) e Y .
So, F=
I.. y
I l x xex
= -- -- e == -
(D -1)
2 2
_ __ _ _ _ _ _ _ _ _ _ _ _ _ _ _ _ _ __ _ _ _ _ M~ll;~
23. (1> 1 I {) 7 . /) I )y O ~
Thus, soluti on is · .I
,. ( 1)7( I J f I) (/J f I )y O
f ~-." wfr + r
(/) f 1)7 ( n l )y O
X So. solution is
impli es = V ·I C
y c,e" f- (cl I Cr)e ,
." So, 2
(!) - 4D + 8 )y IOe" co, l
im!)l ies x = cy+ y 2 C. F. - e 2' ( k cos 2x f- It, sin 2.x)
1

18. Here.,·' = 2,. _.•·


I T
implies 2'' dy = 2' dx and P.l. - I Oe CO<;.t
(D 2 - 4D +R)
2-'' 2'
impli es - - - +c
log 2 log 2 - I Oe' , c.o,;,r
implies ( O + l t - 4<0+ 1!+8
2X- 2Y = c I

19. c/y
X --'-
. 2 3 • 2
IOe-'
dx + sm .V = X Slll y en, r
D 1 - 20 + 5

2 dy cot y
implies cosec y - +2-- = x 2 = I Oe-' - -1- - co, .r
dx X
- 20 +4
Let cot y = 2
- X
dz 2z = - ::, e - -
I - co, t
Thus, - - +- = x2 (D -2)
dx X

implies -dz - -2z = - x 2 = - 5ex D + 2 co, t


1
dx X (D - 4)

= e' (- sin x - :: .:os r


f _3. dx I So, y sin 2.r ) -
So, l.F. = e X =2
= e2x (k 1 cos 2t + k, .r
X (2 cos x - sin x)

. . 1 dy - x + y-3
Hence, solut10n LS. z ·2 =-x + c 25. we have dx - (x + y) + l
X 2
implies z = cot y = -x3 + cx 2 Let 1·= :r- _,
X
1 X xe d\' ,n
20. e Then - = IT.....:....
(D 2 + 3) (D + 1) (D - 1) 8 d, J\

j X d\ ' - I
21. P.I. = - - e cos 2x implies
(D-1) 3 ,h 2\ T l

,I\ J\ - -
~~

= ex~ cos 2x = ex . D4 cos 2x or -


D d\ 2\ + l
D
X l 2\ ' + l )
= _!:_ sin 2x impl ies - - - ,/\ ' - ,fr
8 l3\' - 2)

22. y =x sin x
y' = sin X + X cos X impl ies
J ~3 d \ ' + 1· 9l \ ' 7- '2.\) d \ ' f ,h
y" = cos X + cos X - X sin X
y" + )I = 2 CO S X . . (i ) 2 7 \ t •
implies - V + - In t\ - !.f.\ \
y"' + y ' = - 2 sin x 1 l)
y"" + y " = -= - 2 COS X ... (ii) imp lies 6 (.\ ·I y ) t /11( \ t 1· ~ J) \J 1
,
(i) + (ii) impl ies y"" + )y" I )' u implies 7/11{,h I .II' 2) .h t Cl\'
01FFE!<ENTIAL EQUATION

26. Here IF = eh
So, solution is 719
29. We have I. F. = ef 2 < dx = e x
Y ·eh= f (2e2x + e 2x-x' )dx+c
Hence, solution is

=eh+e f e-(x-lJ' dx+c yex 2 = J2e-x 2 · e _,) dx = 2 dx f


2
implies y =1 + e · e- 2x f e-<x- I)' d.x + ce-2x implies yex = 2x+C

Hence, implies Y = (2x + C ) e- .r-


lim Y(x)
X ➔ oo
=I 30. Here, A E is m 2 + m - 2 = 0
2 or m 2 + 2m - m -· 2 = 0
27. Let L =z then dy dz or m(m + 2) - (m - 2J = 0
2 Y-=-
dx dx or (m + 2) (m - I) = 0
The differential equation becomes or m =-2. m =I
Hence, General soluti on is y = C e 2-' - C .r
dz 1 1
-+2z =x 31. We have
dx
So, I.F. = e2x I ,, 50e: r , : '
P.I. = SO e - , -- ~~
and solution is D 2 +6D+9 2-+6 x 2- 9

32. l
z ·e2x = xe 2x dx = I xe 2x - 1 e 2x +c We have P.I. = - 3--cos (2:r - l l
f 2 D +!
4
I 1
2 ---=-?-- cos (2x - l) == - -- L-0:- ( 2..-r - l l
y I 1 -2x D(-2-) + I -4-D - l
implies z =-=-x-- +ce
2 2 4
1+4D
---c-? cos (2:r - 1)
l -16D-
1
implies y2 = x--+c'e-2 x
2 l + 4D ., .
- - - ~ , co~. ( _.t - 1)
1-16(-2)-
Equation of circles of centered at (I, 0) is given by
(x - I )2 + y2 = ').,2 1
= -[cos (2x- 1) + -l D ,·os \ : .t - l \j
Where A is parameter 65

Differential ing w.r.t. x, we get = ~I ms ( :~.\ - l) - S si n t 2.\ - I) l


6S
2(x - I)+ 2yy' = 0
33. The p-discrimina nt equa ted tl) ze ro m:i~ include the
So, differential equation is
envelope(£) 0111:e, the ~·u.sp-locus ( <.')once and tac- locus
(7) tWil'C:
dy
y -+x =I implies fl'f 2
dx

You might also like